Ch 1- Intro to Accounting and Financial Reporting for Governmental and Not-for-Profit Organizations, Ch2 - Overview of Financial Reporting for State and Local Governments, Ch 3- Modified Accrual Accounting, Chapter 4 MCQ, Ch. 5 | Government and Not F...

Réussis tes devoirs et examens dès maintenant avec Quizwiz!

Which type of pension plan most commonly results in an unfunded actuarial liability? A) Defined benefit plan. B) Defined contribution. C) Both A and B. D) Neither A or B.

A) Defined benefit plan.

In addition to a Statement of Net Assets, what statements are required in the government-wide Financial Statements? A) Statement of Activities. B) Statement of Cash Flows. C)Both of the above. D)Neither of the above.

A) Statement of Activities.

111. When a government is the sponsor of a multi-government investment pool, the government should report the external portion of those trust assets in a(n): A) Pension Trust Fund. B) Investment Trust Fund. C) Private-Purpose Trust Fund. D) Agency Fund.

B) Investment Trust Fund

Which of the following funds is used to account for the external portion of investment pools? A) Special Revenue Fund B) Investment Trust Fund C) Enterprise Fund D) Private-purpose Trust Fund

B) Investment Trust Fund

This fund accounts for activities that produce goods or services to be provided to other departments or other governmental units. A) Enterprise fund. B) Internal Service fund. C) Special Revenue fund. D) Debt Service fund.

B) Internal Service fund.

Reciprocal interfund activity A)Does not include interfund loans B)Is the internal counterpart to exchange transactions C)Includes interfund transfers D)None of the above

B) Is the internal counterpart to exchange transactions

Which part of the financial section of the CAFR is considered "Required Supplementary Information?" A) Auditor's Report B) Management's Discussion and Analysis C) Government-Wide Financial Statements D) Fund Financial Statements

B) Management's Discussion and Analysis

Which of the funds listed below would include capital assets in its Statement of Net Assets? A) Permanent. B) Internal Service. C) Special Revenue. D) None of the above; no funds in governmental accounting have capital asset accounts.

B) Internal Service.

When an activity is created to provide goods or services to provide assistance to other departments or other governmental units, it should be accounted for in the: A) Special Revenue Fund. B) Internal Services Fund. C) Enterprise Fund. D) General Fixed Asset Group.

B) Internal Services Fund.

Activities that produce goods or services to be provided to other departments or other governmental units would be reported in which fund? A) Enterprise fund. B) Internal service fund. C) Agency fund. D) Advance fund.

B) Internal service fund.

______ funds are created when individuals or organizations contribute resources with the agreement that principal and/or income will be used to benefit individuals or nongovernmental organizations. A) Agency B) Investment trust C) Private-purpose trust D) Permanent

C) Private-purpose trust

A statement of Cash Flows is required for which type of Fund? A) Governmental B) Fiduciary C) Proprietary D) All of the above

C) Proprietary

Long-term debt paid from proprietary funds is reported as a liability in A) the proprietary fund Statement of Net Assets. B) the government-wide Statement of Net Assets. C) both A and B. D) neither A nor B.

C) both A and B.

Which of the follow schedules are required to be included in RSI of governments that provide pension trust funds and other post employment benefits: A) Schedule of Employer Contributions. B) Schedule of Funding Progress. C) Statement of Changes in Fiduciary Net Assets. D) A and B.

D) A and B.

The Financial Accounting Standards Board (FASB) is the standard setting organization for which of the following reporting organizations? A) Private not-for-profits B) Public not-for-profits C) Investor owned businesses D) A and C are both correct.

D) A and C are both correct.

The ______ Funds are used when resources are provided primarily through the use of sales and service charges to parties external to the government and it is the intent of the government to measure net profit or loss. A) Special revenue B) General C) Enterprise D) Agency

C) Enterprise

A city government makes its semiannual payment of interest on revenue bonds issued to pay for the construction of additional public transit stations. Which fund would account for the payment? A) Debt service fund B) Capital projects fund C) Enterprise fund D) Internal service fund

C) Enterprise fund

The City of Aurora increased its revenue budget by $75,000 and the appropriation for the Public Works Department was increased by $100,000. What is the journal entry to record this event in the General Fund? A) Estimated Revenues Control 75,000 Fund Balance 25,000 Appropriations Control 100,000 B) Appropriations Control 100,000 Estimated Revenues Control 75,000 Budgetary Fund Balance 25,000 C) Estimated Revenues Control 75,000 Budgetary Fund Balance 25,000 Appropriations Control 100,000 D) Estimated Revenues Control 75,000 Appropriations Control 75,000

C) Estimated Revenues Control 75,000 Budgetary Fund Balance 25,000 Appropriations Control 100,000

When a liability is incurred as authorized by an appropriation, the appropriation is said to be: A) Encumbered B) Appropriated C) Expended D) Expensed

C) Expended

All of the following are budgetary accounts except: A) Estimated Revenues B) Appropriations C) Expenditures D) All of the above

C) Expenditures

Interfund services purchased by the General Fund are recognized as: A) Transfers Out. B) Decreases in Fund Balance. C) Expenditures. D) None of the above.

C) Expenditures.

The GASB is under the oversight of: A) GAO. B) FASAB. C) FAF. D) FASB.

C) FAF.

The _______ sets the accounting and financial reporting standards for both state and local governments and public not-for-profits. A) FASAB B) FASB C) GASB D) AICPA

C) GASB

Which of the following are best described as proprietary funds? I. Internal Service funds II. Pension funds III. Enterprise funds A) I only. B) I and II. C) I and III. D) I, II, and III.

C) I and III.

Capital assets of internal service funds should be reported: A) In the internal service funds only. B) In the government-wide statements only. C) In both the internal service funds and the government-wide statements. D) None of the above, they are not reported

C) In both the internal service funds and the government-wide statements

An endowment to support scholarships would most likely be accounted for in which of the following fund types? A) Agency Fund. B) Investment Trust Fund. C) Private-Purpose Trust Fund. D) None of the Above.

C) Private-Purpose Trust Fund.

The following items were included in the City of Wilson's General Fund expenditures for the year ended June 30, 2015: Computer for City Clerk $ 3,000 Maintenance on heating units for City Hall $ 9,000 How much should be classified as capital assets in Wilson's General Fund balance sheet at June 30, 2015? A) $ -0-. B) $ 3,000. C) $ 9,000. D) $12,000.

A) $ -0-.

The Township of Thomasville's General Fund has the following net resources at year end: • $77,000 of prepaid insurance • $375,000 rainy day fund approved by the township governing board with specific conditions for its use • $2,500 of supplies inventory • $61,000 state grant for snow removal • $150,000 contractual obligations for the purchase of equipment • $200,000 to be used to fund government operations in the future • Outstanding encumbrance of $80,000 for the purchase of furniture & fixtures What would be the total Restricted fund balance? A) $61,000 B) $150,000 C) $200,000 D) $375,000

A) $ 61,000

81. Norton County operated a landfill, and accounted for it as an enterprise fund. The closure and postclosure care costs are estimated to be $15,000,000. It is estimated that the capacity of the landfill is 5 million tons of waste and that waste will be accepted for 10 years. During 2014, 250,000 tons of waste was accepted by the landfill. The charge for closure and postclosure care costs for 2014 would be: A) $ 750,000. B) $1,500,000. C) $ 15,000,000 D) Impossible to determine from the information given.

A) $ 750,000.

136. The amount of capital outlay expenditures reported by the capital projects fund would be: A) $1,980,000. B) $2,000,000. C) $2,040,000. D) $3,000,000.

A) $1,980,000.

The City of Sycamore has investments in bonds. These bonds have an amortized cost of $1,996,000. At year end, the financial press reports a market value of $2,002,000 for these bonds. The original cost of the bonds was $1,992,000. The par value at maturity will be $2,000,000. The amount at which the investments would be reported is: A) $2,002,000 B) $2,000,000 C) $1,996,000 D) $1,992,000

A) $2,002,000

The Township of Thomasville's General Fund has the following net resources at year end: • $77,000 of prepaid insurance • $375,000 rainy day fund approved by the township governing board with specific conditions for its use • $2,500 of supplies inventory • $61,000 state grant for snow removal • $150,000 contractual obligations for the purchase of equipment • $200,000 to be used to fund government operations in the future • Outstanding encumbrance of $80,000 for the purchase of furniture & fixtures What would be the total Unassigned fund balance? A) $200,000 B) $280,000 C) $ 80,000 D) $377,500

A) $200,000

Which of the following is true regarding Government-wide financial statements? A) A Statement of Cash Flows is not required for Government-wide statements. B) Prior year data must be presented C) Depreciation can be reported in the statements or disclosed in footnotes. D) Fiduciary activities are only reported if they qualify as a major fund.

A) A Statement of Cash Flows is not required for Government-wide statements.

The journal entry to record the re-establishment of encumbrances that were outstanding at the close of the prior period would include: A) A credit to Budgetary Fund Balance - Reserve for Encumbrances B) A debit to Budgetary Fund Balance - Reserve for Encumbrances C) A credit to Encumbrance Control D) A and B would both be included in the journal entry

A) A credit to Budgetary Fund Balance - Reserve for Encumbrances

The general ledger journal entry in the General Fund to record the adoption of a budget would include: A) A debit to Estimated Revenues Control B) A credit to Estimated Other Financing Sources Control C) A debit to Budgetary Fund Balance-Reserve for Encumbrances D) A debit to Appropriations Control

A) A debit to Estimated Revenues Control

What basis of accounting would the Enterprise Fund use? A) Accrual B) Modified Accrual C) Cash D) Expended Accrual

A) Accrual

Which of the following statements is true regarding proprietary funds? A) Accruals and deferrals common to business accounting are recorded in proprietary funds. B) Proprietary funds use the modified accrual basis of accounting C) GASB standards require budget-actual reporting for proprietary funds. D) The purchase of a capital asset is recorded as an expenditure in proprietary funds.

A) Accruals and deferrals common to business accounting are recorded in proprietary funds.

Internal service funds account for A) Activities that produce goods or services to be provided to other governmental units on a cost reimbursement basis. B) Activities that produce goods or services to be provided to outside consumers units on a cost reimbursement basis. C) Both A and B. D) None of the above.

A) Activities that produce goods or services to be provided to other governmental units on a cost reimbursement basis.

A fund that is used to account for assets held by a government acting as agent for one or more other governments units or for individuals or private organizations is a(n): A) Agency fund B) Private-Purpose Trust Fund C) Investment Trust Fund D) Pension Trust Fund

A) Agency fund

Which of the following statements is not correct with respect to Agency Funds? A) Agency funds use modified accrual accounting B) Assets accounted for in an agency fund belong to the party or parties for which the government acts as agent. C) Assets are offset by an equal amount of liabilities. D) Assets and liabilities are recognized at the time the government becomes responsible for the assets.

A) Agency funds use modified accrual accounting

Which of the following statements is not true regarding agency funds? A) Agency funds use modified accrual accounting. B) Agency funds report only assets and liabilities; no fund equities, revenues, or expenditures are recorded. C) Agency funds are reported in the fiduciary funds Statement of Fiduciary Net Assets. D) None of the above; all are true.

A) Agency funds use modified accrual accounting.

Which is true regarding the basis of accounting for fiduciary funds? A) All are accounted for using the same basis of accounting and measurement focus as proprietary funds. B) Agency funds are accounted for using the same basis of accounting and measurement focus as governmental funds, trust funds are accounted for using the same basis of accounting and measurement focus as proprietary funds. C) Trust funds are accounted for using the same basis of accounting and measurement focus as governmental funds, and agency funds are accounted for using the same basis of accounting and measurement focus as proprietary funds. D) All are accounted for using the same basis of accounting and measurement focus as governmental funds.

A) All are accounted for using the same basis of accounting and measurement focus as proprietary funds.

The difference between an expenditure and an encumbrance is? A) An expenditure is the actual amount of an authorized liability and encumbrances are estimates. B) An encumbrance is an expended expenditure. C) Enterprise funds record expenditures, but do not record encumbrances. D) Expenditures are estimates and encumbrances are not.

A) An expenditure is the actual amount of an authorized liability and encumbrances are estimates.

Governments that contribute to single employer and agent multiple-employer plans compute annual pension cost as the _________? A) Annual required contribution. B) Normal cost. C) Unfunded actuarial liability. D) Net pension obligation.

A) Annual required contribution.

The legal authorization for the administrators of the governmental unit to incur liabilities during the budget period for purposes specified in the appropriations statute or ordinance and not to exceed the amount specified for each purpose is a (an): A) Appropriation B) Encumbrance C) Other financing source/use D) Expenditure

A) Appropriation

What is the first step a government takes in acquiring goods or services through the General Fund? A) Appropriation B) Expenditure C) Requisition order D) Encumbrance

A) Appropriation

Proprietary funds: A) Are required to present a reconciliation between the Statement of Revenues, Expenses, and Changes in Fund Net Assets and the Cash Flow Statement B) Are required to present budget-actual statements in the fund statements C) Are not required to accrue interest due more than 30 days after the end of the fiscal year D) Are required to present a Statement of Revenues and Expenses and Balance sheet.

A) Are required to present a reconciliation between the Statement of Revenues, Expenses, and Changes in Fund Net Assets and the Cash Flow Statement

How should customer deposits held for water meters that cannot be spent for operating purposes be classified? A) As restricted cash in an Enterprise Fund. B) As both cash and a liability in an Agency fund. C) As Nonspendable fund balance in a Permanent Fund. D) Any of the above are acceptable alternatives.

A) As restricted cash in an Enterprise Fund.

Which type of pension plan is required to pay out a certain sum, generally based on a percentage of salary upon retirement and the number of years of service? A) Defined Benefit B) Defined Contribution C) Contributory D) Noncontributory

A) Defined Benefit

Which of the following is not true regarding the Statement of Cash Flows for proprietary funds? A) At the bottom of the statement, net income is reconciled to cash flows from operations. B) Interest and dividends received are recorded as investing activities. C) The direct method is required for reporting cash flows from operations. D) Four categories of cash flows are used.

A) At the bottom of the statement, net income is reconciled to cash flows from operations.

Agency fund assets and liabilities are to be recognized : A) At the time the government becomes responsible for the assets. B) When they are available and measurable. C) Only in the government-wide financial statements. D) When the earnings process is complete and collection is reasonably assured.

A) At the time the government becomes responsible for the assets.

Under modified accrual accounting, revenues are recognized when measurable and A) Available. B) Collected. C) Earned. D) Expenditures have been made.

A) Available

A certain government passed its budget for the fiscal year ended December 31, 2014. Estimated Revenues amounted to $13,000,000; Appropriations amounted to $12,000,000; Estimated Other Financing Uses amounted to $700,000; and Estimated Other Financing Sources amounted to $200,000. In the budgetary entry (combining entry): A) Budgetary Fund Balance would be credited for $500,000. B) Budgetary Fund Balance would be debited for $500,000. C) Budgetary Fund Balance would be debited for $1,000,000. D) Budgetary Fund Balance would be credited for $1,500,000.

A) Budgetary Fund Balance would be credited for $500,000.

Which of the following statements is correct with respect to GASB Statement No. 53 which establishes reporting requirements for governments entering into derivative instruments? A) Changes in the value of hedge derivatives are reported in the Statement of Net Assets. B) If a derivative is an investment derivative, the changes in the value of the derivative are deferred and reported in the statement of net assets. C) Statement No. 53 does not apply to proprietary or fiduciary funds. D) Statement No. 53 applies to government financial statements prepared using the modified accrual basis of accounting.

A) Changes in the value of hedge derivatives are reported in the Statement of Net Assets.

Clinton County maintains an investment trust fund for the investments of governments within its borders. All the investments had determinable fair values. Which of the following is true regarding investment trust funds and investments in general? A) Clinton County would report the investments of the other governments at fair value in the investment trust funds. B) Clinton County would report its own investments at fair value in the investment trust funds. C) Both of the above. D) Neither of the above.

A) Clinton County would report the investments of the other governments at fair value in the investment trust funds.

Which of the following statements is correct with respect to GASB Statement No. 54 regarding committed funds? A) Committed funds are those that are designated as committed through ordinance or resolution by the government's highest level of authority. B) These constraints are easily changed by administrators. C) This is constraint is imposed by law and may not be changed without amendment of the law. D) Committed funds may be redesignated to uncommitted in the event of a budget shortfall.

A) Committed funds are those that are designated as committed through ordinance or resolution by the government's highest level of authority.

When the materials and supplies are received and the invoice is more than the related purchase order, this entry should be made: A) Debit Expenditures Control for the amount of the invoice. B) Debit Expenditures Control for the amount of the purchase order. C) Credit Accounts Payable for the amount of the purchase order. D) Credit Encumbrances Control for the amount of the invoice.

A) Debit Expenditures Control for the amount of the invoice.

Expenditure classifications by function includes all of the following except A) Debt Service B) General Government C) Public Safety D) Highways & Streets

A) Debt Service

To record the budget for the General Fund A) Estimated Revenue Control is Debited B) Expenditure Control is Debited C) Appropriations Control is Debited D) Revenue Control is Debited

A) Estimated Revenue Control is Debited

When a computer which is to be used by an activity accounted for by the General Fund of a governmental unit is received, it should be recorded in the General Fund as a(an): A) Expenditure. B) Appropriation. C) Encumbrance. D) Capital asset.

A) Expenditure.

Which of the following results in a decrease in fund balance on a governmental fund's Balance Sheet? A) Expenditures B) Transfers Received C) Encumbrance D) Both A and B

A) Expenditures

General Fund resources were expended in the amount of $185,000 to purchase a new fire truck. The General Fund would debit: A)Expenditures Control. B)Capital assets -Motor Vehicles. C)Both (a) and (b) above. D)None of the above; no entry is made in the General Fund when a motor vehicle is purchased because only current financial resources are recorded in the General Fund.

A) Expenditures Control.

Investment Trust Funds are restricted to: A) External investment pools. B) Open-end mutual funds. C) Debt securities. D) All the above.

A) External investment pools

Which of the following organizations has authority to establish accounting and financial reporting standards for the federal government? A) Federal Accounting Standards Advisory Board. B) Office of Management and Budget. C) Governmental Accounting Standards Board. D) None of the above; no one has been granted authority to set standards for the federal government

A) Federal Accounting Standards Advisory Board.

Which of the following is true regarding fiduciary funds? A) Fiduciary funds use the economic resources measurement focus and accrual accounting. B) Fiduciary funds include pension (and other employee benefit) trust, investment trust, agency, and permanent funds. C) Both of the above. D) Neither of the above.

A) Fiduciary funds use the economic resources measurement focus and accrual accounting.

Under modified accrual accounting, budgetary authority expires at year end and the balances of Revenues and Expenditures are closed to A) Fund Balance B) Net Assets C) Other Financing Uses D) Other Financing Sources

A) Fund Balance

The asset, equipment, used by a department accounted for by the General Fund of a governmental unit should be reported in the: A) Government-wide statements. B) General Fund. C) Both (a) and (b). D) None of the above.

A) Government-wide statements.

Moving cash from the General Fund to a debt service fund for payment of debt service is an example of a (an) A) Interfund transfer. B) Interfund loan. C) Interfund reimbursement. D) Quasi-external transaction.

A) Interfund transfer.

Which of the following funds use accrual accounting? A) Internal Service Fund. B) Special Revenue Fund. C) Permanent Fund. D) Both A & B.

A) Internal Service Fund.

Which of the following is correct with respect to Internal Service Funds? A) Internal service funds use accrual accounting and the economic resource measurement focus B) Net Assets (fund equity) are to be reported in two categories: assigned and unassigned C) Internal service funds account for long-term debt but not capital assets D) All of the above are false statements

A) Internal service funds use accrual accounting and the economic resource measurement focus

Which of the following would not be true regarding internal service funds? A) Internal service funds use the current financial resources measurement focus and accrual basis of accounting. B) Examples of internal service funds would include self-insurance funds, motor pool funds, and print shop funds. C) In the basic financial statements, internal service funds are reported in the proprietary funds financial statements. D) None of the above; all are true.

A) Internal service funds use the current financial resources measurement focus and accrual basis of accounting.

What are the three major sections of the Comprehensive Annual Financial Report? A) Introductory Section, Financial Section, Statistical Section B) Introductory Section, Financial Section, Management Discussion and Analysis. C) Introductory Section, Financial Section, Budgetary Comparison D) Introductory Section, Financial Section, Reporting Infrastructure

A) Introductory Section, Financial Section, Statistical Section

A county treasurer maintains an investment pool in which several different towns in the county hold investments. Where should the towns' investments be recorded? A) Investment Trust Fund. B) Agency Fund. C) Private-purpose Trust Fund. D) None of the above.

A) Investment Trust Fund.

Which of the following is true regarding accounting for investments by state and local governmental units? A) Investments in all securities with determinable fair values (including bonds held to maturity) are to be reported at fair value. B) Realized gains and losses are to be reported separately from unrealized gains and losses in the Statement of Changes in Fiduciary Net Assets. C) Both of the above. D) Neither A nor B above.

A) Investments in all securities with determinable fair values (including bonds held to maturity) are to be reported at fair value.

The categories of Net assets for a proprietary fund are classified within which of the following categories? A) Net Assets Invested in Capital Assets Net of Related Debt, Restricted Net Assets and Unrestricted Net Assets B) Spendable Net Assets and Spendable Net Assets C) Committed Net Assets, Assigned Net Assets and Unassigned Net Assets D) Restricted Fund balance, Committed Fund balance, Assigned Fund Balance, and Nonspendable Fund Balance

A) Net Assets Invested in Capital Assets Net of Related Debt, Restricted Net Assets and Unrestricted Net Assets

A donor pledged $200,000 to the fund raising drive of a local government to assist its police officers in obtaining the latest technology. The pledge was made on July 16, 2011 but was conditioned on the government raising an additional $200,000 from other donors. By the fiscal year-end of June 30, 2012, the local government had raised only $5,000 from other donors. What entry would be made for the initial pledge by the local government during the year ended June 30, 2012? A) No entry is required until the condition is fully met. B) Debit a receivable and credit deferred inflows. C) Debit a receivable and credit Fund Balance. D) Debit a receivable and credit revenue

A) No entry is required until the condition is fully met

66. A transaction in which a government gives or receives value without directly receiving or giving equal value in exchange is known as a(n): A) Non-exchange Transaction. B) Fair Exchange. C) Equity Transaction. D) Not Fair Market Exchange transaction.

A) Non-exchange Transaction.

When would a special revenue fund be deemed to have satisfied the eligibility requirement of a reimbursement-type federal grant, under GASB Statement 33?

A) Only as work is completed for a project.

Which of the following is not an example of the function of an Agency Fund? A) Payment of pension benefits to retired government employees B) Collection of local government sales taxes by the state government C) Collection of property taxes for the school district by an elected county government official. D) Periodic distribution by the state government of county sales taxes

A) Payment of pension benefits to retired government employees

Which of the following results in an encumbrance? A) Place a purchase order. B) Receive goods previously ordered. C) Receive an invoice on goods previously received. D) Payment of an invoice previously received.

A) Place a purchase order.

When a contributor and a government agree that the principal and/or income of trust assets are for the benefit of individuals, private organizations, or other governments, which fund is formed? A) Private-purpose trust. B) Agency. C) Investment trust. D) Pension trust.

A) Private-purpose trust.

Which of the following is true with respect to rainy day funds? A) Rainy day funds are classified as committed if they are created by a resolution or ordinance that identifies the specific circumstances under which the resources may be expended. B) If rainy day funds are available "in emergencies" or periods of "revenue shortfalls" they are classified as committed. C) Both A and B. D) Neither A nor B.

A) Rainy day funds are classified as committed if they are created by a resolution or ordinance that identifies the specific circumstances under which the resources may be expended.

When preparing the Statement of Cash Flows for the Proprietary Funds, cash flows from noncapital financing activities include all of the following EXCEPT: A) Receipts from customers. B) Proceeds and repayment of debt not clearly related to capital outlay. C) Grants received from and paid to other governments for noncapital purposes. D) Transfers to and from other funds

A) Receipts from customers.

Which of the following is an environmental difference between governments and for-profit business enterprises? A) Relationship with stakeholders B) Donor restricted assets C) Fiduciary responsibilities D) Both B and C

A) Relationship with stakeholders

The city of Canandaigua receives proceeds from the sale of land, the transaction is considered to be a special item. The proceeds are: A) Reported separately after other financing sources and uses. B) Reported as a revenue. C) Reported as an item that changes the Fund Balance. D) Not recorded but the gain on the sale is.

A) Reported separately after other financing sources and uses.

A December 2015 year-end government should record which of the following as expenditures: A) Salaries Payable due January 20, 2015. B) Bond Payable that matures January 30, 2015. C) Bond Payable that matures February 14, 2015. D) Both A & B.

A) Salaries Payable due January 20, 2015.

Which of the following is not true regarding enterprise funds? A) Similar to commercial businesses, enterprise funds equity accounts include both contributed capital and retained earnings. B) A Statement of Revenues, Expenses, and Changes in Fund Net Assets would be the primary operating statement for enterprise funds. C) Enterprise funds' Cash Flow Statements could have up to four categories. D) None of the above; all are true.

A) Similar to commercial businesses, enterprise funds equity accounts include both contributed capital and retained earnings.

Which of the following fund types uses modified accrual accounting? A) Special Revenue B) Internal Service C) Investment Trust D) Enterprise

A) Special Revenue

The ______ Fund accounts for and reports the proceeds of specific revenue sources that are restricted or committed to expenditure for specified purpose other than debt service or capital projects. A) Special revenue B) General C) Enterprise D) Agency

A) Special revenue

Which of the following is true with respect to Special revenue funds? A)Special revenue funds are used when it is desirable to provide separate reporting of resources that are restricted or committed as to expenditure for purposes other than debt service or capital projects. B)Special revenue funds are used when it is desirable to provide separate reporting of resources that are designated for specific purposes. C)Special revenue funds are used when it is desirable to provide separate reporting for funds provided by other governments to support capital projects. D)Both B and C

A) Special revenue funds are used when it is desirable to provide separate reporting of resources that are restricted or committed as to expenditure for purposes other than debt service or capital projects

Which of the following is an example of an Internal Service Fund? A) State Risk Management Fund B) County Water and Sewer Utilities. C) State Lotteries. D) Municipal Golf Courses.

A) State Risk Management Fund

In addition to a Statement of Net Assets and Statement of Revenues, Expenses and Changes in Fund Net Assets, which of the following statements is (are) required for proprietary funds? A) Statement of Cash Flows. B) Budgetary Comparison Schedule. C) Both A and B above D) Neither A nor B above

A) Statement of Cash Flows.

Which of the following statement(s) are not included in the proprietary funds: A) Statement of Revenues, Expenses, and Changes in Net Assets B) Statement of Cash Flows C) Statement of Net Assets D) All of the above are included

A) Statement of Revenues, Expenses, and Changes in Net Assets

Which financial statements are required for proprietary funds? A) Statement of net assets; Statement of revenues, expenses, and changes in fund net assets; Statement of cash flows. B) Budgetary comparison schedule; Statement of revenues, expenditures, and changes in fund balance; Statement of net assets. C) Statement of net assets; Statement of revenues, expenditures, and changes in fund balances; Statement of cash flows. D) Statement of net assets; Statement of revenues, expenses, and changes in fund net assets;

A) Statement of net assets; Statement of revenues, expenses, and changes in fund net assets; Statement of cash flows.

The City of DeKalb contributes to three pension plans: (1) a statewide pension plan for nonuniformed personnel, (2) a locally administered plan for police officers, and (3) a locally administered plan for firefighters. The City is trustee for the plans for police officers and firefighters. Which of the following is true? A) The City would maintain trust funds for the police and fire plans but not the statewide plan. B) When making contributions from General Fund resources to the police and fire plans, the General Fund would debit Other Financing Sources-Transfers Out and the Pension Trust Fund would credit Transfers In. C) Both of the above. D) Neither of the above.

A) The City would maintain trust funds for the police and fire plans but not the statewide plan.

The operations of agency funds will be included in which of the following statements? A) The fiduciary funds Statement of Fiduciary Net Assets B) The Statement of Changes in Fiduciary Net Assets C) The Government-wide Financial Statements D) All of the above

A) The fiduciary funds Statement of Fiduciary Net Assets

According to GASB Statement 31, Accounting and Financial Reporting for Certain Investments and for External Investment Pools, investments of internal investment pools are to be reported in: A) The funds providing the resources for the investments. B) An investment trust fund. C) An agency fund. D) Either A and B.

A) The funds providing the resources for the investments.

According to GASB Statement No. 54 an unassigned fund balance is A) The residual category for the General Fund. B) The only category of Fund balance that may not be reported as a negative number. C) The designation given for rainy day funds. D) None of the above

A) The residual category for the General Fund.

Which of the following is true about the combining financial statements? A) They are used whenever a non-major column is used in one of the fund financial statements B) The total column in the combining statements is not always the same as the non-major funds column in the basic financial statements C) Both of the above D) Neither of the above

A) They are used whenever a non-major column is used in one of the fund financial statements

A reconciliation from Enterprise funds statements to government-wide statements is not necessary because ... A) They both use the same measurement focus and basis of accounting (economic resources, accrual) B) They both use the same measurement focus and basis of accounting (current financial resources, modified accrual) C) They use a different measurement focus and basis of accounting. D) They use a different measurement focus and basis of accounting.

A) They both use the same measurement focus and basis of accounting (economic resources, accrual)

The term special item is defined as: A) Unusual or infrequent but within management's control. B) Frequent and unusual but within management's control. C) Frequent and unusual and not within management's control. D) Unusual or infrequent and not within management's control.

A) Unusual or infrequent but within management's control.

Under GASB Statement 34, enterprise funds must be used in which of the following circumstances? A) When debt is backed solely by fees and charges. B) When services are provided to other governmental units for a fee. C) When the activities are expected to make a profit to be used to fund other government programs. D) None of the above; enterprise funds are optional.

A) When debt is backed solely by fees and charges.

GASB Statement 34 requires enterprise funds to be used under which of the following circumstances? A) When the legal requirement exists that the cost of providing services for an activity, including capital costs, be recovered through fees or charges. B) When debt is backed by the full faith and credit of the government issuing it. C) When a government is restricted by grant covenants to cover the cost of providing services for an activity. D) All of the above.

A) When the legal requirement exists that the cost of providing services for an activity, including capital costs, be recovered through fees or charges.

Indicate which of the following use Modified Accrual Accounting? -Capital Projects Fund -Government-wide Financial Statements -Special Revenue Fund -Internal Service Fund A) Yes No Yes No B) No Yes No Yes C) Yes No Yes Yes D )Yes Yes No No

A) Yes No Yes No

Assume estimated revenues exceed appropriations. When the budget for the General Fund is recorded, the journal entry will include: A) a credit to Budgetary Fund Balance. B) a credit to Estimated Revenues Control. C) a debit to Appropriations Control. D) a credit to Budgetary Fund Balance -- Reserve for Encumbrances.

A) a credit to Budgetary Fund Balance.

In the government-wide statements, enterprise funds are a(n) ______ activity, whereas Internal Service funds are typically a(n) ______activity. A) business-type; governmental. B) governmental; business-type. C) restricted; unrestricted. D) unrestricted; restricted.

A) business-type; governmental.

The ___________ is the government's official annual report prepared and published as a matter of public record. A) comprehensive annual financial report B) governmental annual financial report C) independent auditor's report D) complete audited financial report

A) comprehensive annual financial report

In a budgetary entry, if Appropriations Control exceeds Estimated Revenues Control, the excess would be: A) debited to Budgetary Fund Balance. B) credited to Budgetary Fund Balance. C) credited to Fund Balance-Unassigned. D) credited to Fund Balance-Reserved for Encumbrances.

A) debited to Budgetary Fund Balance.

The term "proprietary funds" applies to: A) enterprise and internal service funds only. B) all funds that use accrual accounting. C) enterprise, internal service, and private-purpose trust funds. D) none of the above answers are correct.

A) enterprise and internal service funds only.

Long-term debt of a governmental unit would be reported in the: A) government-wide financial statements and proprietary fund financial statements. B) government-wide financial statements only. C) debt service funds of governmental fund financial statements and government-wide financial statements. D) notes to the financial statements only.

A) government-wide financial statements and proprietary fund financial statements.

The Revenues account of a government is credited when: A) property taxes are levied. B) property taxes are collected. C) the budget is recorded in the accounts. D) budgetary accounts are closed at the end of the year.

A) property taxes are levied.

Which of the following is true regarding the government-wide Statement of Net Position? A)Discretely presented component units are included in a separate column (or columns). B)Net assets are broken down into three categories: invested in capital assets, reserved, and unreserved. C)Both of the above. D)Neither of the above.

A)Discretely presented component units are included in a separate column (or columns).

Which of the following is true regarding the government-wide statements? A)The government-wide statements include a Statement of Net Position and a Statement of Activities. B)The governmental activities portion of the government-wide statements is prepared using the current financial resources measurement focus and modified accrual basis of accounting. C)Both of the above. D)Neither of the above.

A)The government-wide statements include a Statement of Net Assets and a Statement of Activities.

Which of the following is true regarding the fiduciary fund financial statements? A)Fiduciary fund financial statements include the Statement of Fiduciary Net Assets and Statement of Changes in Fiduciary Net Assets. B)Fiduciary fund financial statements are prepared using the current financial resources measurement focus and modified accrual basis of accounting. C)Fiduciary fund financial statements reflect equity as reserved and unreserved. D)Agency funds are reported only on the Statement of Changes in Fiduciary Net Assets.

A)Fiduciary fund financial statements include the Statement of Fiduciary Net Assets and Statement of Changes in Fiduciary Net Assets.

Which of the following is true regarding fiduciary fund statements? A)Fiduciary fund statements include the Statement of Fiduciary Net Assets and the Statement of Changes in Fiduciary Net Assets. B)Fiduciary fund statements are prepared using the current financial resources measurement focus and modified accrual basis of accounting. C)Both of the above. D)Neither of the above.

A)Fiduciary fund statements include the Statement of Fiduciary Net Assets and the Statement of Changes in Fiduciary Net Assets.

Which of the following is true regarding fiduciary funds? A)Fiduciary funds are reported in the fiduciary fund financial statements but not in the government-wide financial statements. B)Agency funds would be reported in both the Statement of Fiduciary Net Assets and the Statement of Changes in Fiduciary Net Assets. C)Both of the above. D)Neither of the above.

A)Fiduciary funds are reported in the fiduciary fund financial statements but not in the government-wide financial statements.

Which of the following are the governmental funds? A)General, special revenue, debt service, capital projects, permanent. B)General, special revenue, debt service, capital projects, private purpose. C)General, special revenue, debt service, capital projects, internal service. D)None of the above.

A)General, special revenue, debt service, capital projects, permanent.

Level "A" GAAP for Cook County Hospital, a public hospital, would be established by the: A)Governmental Accounting Standards Board. B)Financial Accounting Standards Board. C)Hospital Financial Management Association. D)American Institute of Certified Public Accountants

A)Governmental Accounting Standards Board.

Which of the following is true regarding financial reporting of governmental funds? A)Governmental funds include the General Fund, special revenue funds, capital projects funds, debt service funds, and permanent funds. B)The Fund Balance of governmental funds reports fund balance as reserved and unreserved. C)Both of the above. D)Neither of the above.

A)Governmental funds include the General Fund, special revenue funds, capital projects funds, debt service funds, and permanent funds.

Which of the following is true regarding fund classifications? A)Governmental funds include the General, special revenue, debt service, capital projects, and permanent. B)Proprietary funds include enterprise, internal service, and private-purpose. C)Both of the above. D)None of the above.

A)Governmental funds include the General, special revenue, debt service, capital projects, and permanent.

Which of the following is true regarding the financial reporting entity of a state or local government? A)Many component units are discretely presented, rather than blended. B)A school district could never be a primary government. C)Only a primary government is reported in the government-wide statements. D)All of the above are true.

A)Many component units are discretely presented, rather than blended.

A city government collects local option sales taxes legally restricted to pay for the hiring of teachers for hearing impaired school children. Which fund should account for the receipt of the sales taxes? A)Special revenue fund B)Capital projects fund C)Private-purpose trust fund D)General fund

A)Special revenue fund

Which of the following is true regarding the government-wide Statement of Activities? A)The Statement of Activities includes depreciation expense. B)Transfers between governmental and business type activities are eliminated and do not appear on the government-wide Statement of Activities. C)In the government-wide Statement of Activities, "special items" are those items that are both unusual and infrequent and are not under the control of management. D)Taxes levied for specific functions may be reported as program revenue.

A)The Statement of Activities includes depreciation expense.

Which of the following is true regarding the Comprehensive Annual Financial Report? A)The basic financial statements consist of the government-wide, the governmental fund, the proprietary fund, and the fiduciary fund financial statements (and the notes to the financial statements). B)The government-wide financial statements include the Statement of Net Assets, the Statement of Activities, and the Statement of Cash Flows. C)Both of the above. D)Neither of the above.

A)The basic financial statements consist of the government-wide, the governmental fund, the proprietary fund, and the fiduciary fund financial statements (and the notes to the financial statements).

Which of the following is true regarding the government-wide Statement of Net Position? A)The government-wide Statement of Net Position reflects capital assets, net of accumulated depreciation, for both governmental and business-type activities. B)The government-wide Statement of Net Position must be prepared in a classified format; that is, both assets and liabilities must be separated between current and long-term categories. C)The government-wide Statement of Net Position includes all resources entrusted to the government; including governmental, proprietary, and fiduciary. D)A reporting entity (primary government plus component units) total column is required.

A)The government-wide Statement of Net Position reflects capital assets, net of accumulated depreciation, for both governmental and business-type activities.

Which of the following is true regarding the government-wide financial statements? A)The government-wide financial statements include the Statement of Net Assets and the Statement of Activities. B)The government-wide financial statements are prepared using the current financial resources measurement focus and the modified accrual basis of accounting. C)Both of the above. D)Neither of the above.

A)The government-wide financial statements include the Statement of Net Assets and the Statement of Activities.

Which of the following is true regarding the proprietary fund financial statements? A)The proprietary fund financial statements include the Statement of Net Assets, the Statement of Revenues, Expenses, and Changes in Fund Net Assets, and the Statement of Cash Flows. B)The proprietary funds include enterprise, internal service, and permanent funds. C)Both of the above. D)Neither of the above

A)The proprietary fund financial statements include the Statement of Net Assets, the Statement of Revenues, Expenses, and Changes in Fund Net Assets, and the Statement of Cash Flows.

Under GASB rules for the financial reporting entity: A)component units are included if the primary government is financially accountable for their operations. B)counties are component units of the State Government. C)blended and discretely presented component units are to be reported in government-wide financial statements but not in fund financial statements. D)component units must be reported in columns (discrete presentation) separate from the funds of a primary government.

A)component units are included if the primary government is financially accountable for their operations.

Which of the following should be accounted for in a permanent fund:

A. A gift of $1,000,000 to a city, to be invested permanently with the proceeds to be used to maintain the city war memorials

Which of the following is not one of the major sections of the Comprehensive Annual Financial Report (CAFR)? A. Budgetary B. Financial C. Introductory D. Statistical

A. Budgetary

Which of the following is true regarding capital projects funds?

A. Capital projects funds are considered to be governmental funds.

When a purchase order is issued under a Capital Projects fund, how should the transaction be recorded?

A. Debt Encumbrances and credit Budgetary Fund Balance -- Reserve for Encumbrances

Which of the following statements is FALSE?

A. Debt service funds are required to report accrued interest payable as current liabilities.

The FASAB has the authority to establish accounting and financial reporting standards for: A. Federal government B. Investor owned business C. State and local governments D. Public not-for-profits

A. Federal government

Which of the following would not be included in FASB Statement of Financial Accounting Concepts #4, which identifies the information needs of the users of non-business financial statements? A. Format and content of financial statements B. Assessing services provided by a non-business organization and its ability to continue to provide those services C. Assessing managements' stewardship and performance d. Evaluation of an organization's resources, obligations and effects of changes in net resources

A. Format and content of financial statements

Which fund category must change from modified accrual to accrual basis when preparing government-wide financial statements? A. Governmental B. Proprietary C. Fiduciary D. A & B only

A. Governmental

Which of the following is true regarding accounting for investments of permanent funds?

A. Investments with determinable fair value must be reported at fair value

A transfer from the General Fund to a debt service fund to make annual payments on principal and interest would be recorded in the debt service fund as a (an):

A. Other Financing Source

With respect to Debt Service Funds, which of the following is true?

A. Payments under capital lease obligations may be reported in debt service funds.

Grant proceeds received from the state for a capital project would be recorded in a capital projects fund of a city government as a (an):

A. Revenue

Debt service funds use the same measurement focus and basis of accounting as:

A. Special Revenue Funds

When a capital project has been constructed entirely with bond proceeds and funds are left over at the end of the project, which of the following would normally take place?

A. The funds would be transferred to a debt service fund; the capital projects fund would debt Other Financing Uses - Transfers Out and the debt service fund would credit Other Financing Sources - Transfers In.

Which of the following statements is FALSE?

A. Unexpended intergovernmental grants and taxes dedicated to capital improvements in a capital projects fund are likely to be classified as Assigned Fund Balance.

Proprietary funds use the: A. economic resources measurement focus and accrual basis of accounting. B. current financial resources measurement focus and modified accrual basis of accounting C. economic resources measurement focus and modified accrual basis of accounting. D. current financial resources measurement focus and accrual basis of accounting.

A. economic resources measurement focus and accrual basis of accounting.

Which of the following sections of the CAFR is audited? A) Introductory section B) Financial section C) Statistical section D) Management Discussion and Analysis section

B) Financial section

The General Fund of the City of Lexington approved a tax levy for the calendar year 2015 in the amount of $2,000,000. Of that amount, $30,000 is expected to be uncollectible. During 2015, $1,750,000 was collected. During 2016, $100,000 was collected during the first 30 days, $50,000 was collected during days 31-60, and $70,000 was collected during the days 61-90. During the post-audit, you discovered that the City showed $2,000,000 in revenues. How much revenue should the City recognize in 2015 from this tax levy? A) $ 1,850,000 B) $ 1,900,000 C) $ 1,920,000 D) $ 2,000,000

B) $ 1,900,000

Montgomery County operates a landfill as an enterprise fund. The closure and postclosure care costs are estimated to be $24 million. It is estimated that the capacity of the landfill is 12 million tons of waste and that waste will be accepted for 4 years. During 2014, 2.9 million tons of waste was accepted. The charge for closure and postclosure care costs for 2014 would be: A) $ 6 million. B) $ 5.8 million. C) $ 4.5 million. D) $ 3 million

B) $ 5.8 million.

The Fire Department of a given city received an appropriation in the amount of $15,000,000 for the fiscal year ended June 30, 2014. During the month ended July 31, 2013, the following transactions occurred: (a) purchase orders were issued in the amount of $650,000; (b) purchase orders, related to (a) above, were filled in the amount of $610,000; the related invoice amount was $595,000; invoices were paid in the amount of $550,000; (c) salaries were accrued and paid in the amount of $600,000. The balance available for the fire department as of July 31, 2013 is: A) $13,740,000. B) $13,765,000. C) $13,800,000. D) $13,840,000.

B) $13,765,000.

The City of Park Ridge reported appropriations in the amount of $17,000,000 for its General Fund for the fiscal year ended December 31, 2014. During that year, expenditures amounted to $16,200,000 (related to current year purchase orders). In addition, $600,000 in encumbrances had been issued this year, but not filled. Park Ridge follows the budgetary procedure where encumbrances do not lapse. The amount that Park Ridge would report for expenditures (and encumbrances, if applicable) in its 2014 General Fund Statement of Revenues, Expenditures, and Changes in Fund Balances (GAAP Basis) would be: A) $600,000. B) $16,200,000. C) $16,800,000. D) Not enough information is provided.

B) $16,200,000.

The City of Park Ridge reported appropriations in the amount of $17,000,000 for its General Fund for the fiscal year ended December 31, 2014. During that year, expenditures amounted to $16,200,000 (related to current year purchase orders). In addition, $600,000 in encumbrances had been issued this year, but not filled. Park Ridge follows the budgetary procedure where encumbrances do not lapse. In its 2014 Budgetary Comparison Schedule, the amount Park Ridge reports for expenditures (and encumbrances, if applicable) would be: A) $16,200,000. B) $16,800,000. C) $17,000,000. D) Not enough information is provided.

B) $16,800,000.

The tax agency fund of Orange County collected $7,000,000 for a school district, $3,000,000 for the county General Fund, and $5,000,000 for a municipality located in the County. County General Fund employees handle the collections, and a 2.5 percent collection fee is charged all units except the county. The total amount of revenue recognized by the county General Fund would be: A) $ 390,000. B) $3,300,000. C) $3,390,000. D) $15,300,000.

B) $3,300,000.

A local government was awarded a federal grant in the amount of $950,000 to provide for a summer youth employment program for young people. The grant was a reimbursement grant, and a notification of the grant award was received on Apri1 30, 2015. The local government expended the resources as follows: • June 2015: $400,000; • July 2015: $300,000; • August 2015: $250,000. The federal government sent the funds in the month following the expenditure. The local government would recognize revenues for the fiscal year ended June 30, 2015 in which amount? A) $ -0-. B) $400,000. C) $700,000. D) $950,000.

B) $400,000.

The City of Pine Cove levied property taxes for the fiscal year ended June 30, 2015, in the amount of $10,000,000. It is estimated that 2% will not be collected. During the year ended June 30, 2015, $8,500,000 in property taxes was collected. It is estimated that $550,000 will be collected during the next 60 days, $765,000 will be collected after 60 days, and $185,000 will not be collected. What is the maximum amount Lonesome Pines can recognize as property tax revenue for the fiscal year ended June 30, 2015, in its Governmental Funds Statement of Revenues, Expenditures, and Changes in Fund Balances, assuming there were no unpaid property taxes at the end of the previous year? A) $8,500,000. B) $9,050,000. C) $9,815,000. D) $10,000,000.

B) $9,050,000.

The general ledger journal entry in the General Fund to record actual expenditures and reversal of the associated encumbrance would include: A) A debit to Encumbrance Control B) A credit to Encumbrance Control C) A debit to Accounts payable D) A credit to Budgetary Fund Balance-Reserve for Encumbrances

B) A credit to Encumbrance Control

The journal entry to record the property tax levy for a municipality would include: A) A debit to Bad Debts Expense B) A credit to Estimated Uncollectible Current Taxes C) A credit to Taxes Receivable - Current D) A and B would both be included in the journal entry

B) A credit to Estimated Uncollectible Current Taxes

The entry to re-establish encumbrances related to purchase orders from the prior year would include

B) A debit to Encumbrances Control

The journal entry to record the budget of a Special Revenue Fund would include: A) A debit to appropriations Control B) A debit to Estimated Revenues Control C) A credit to Estimated Revenues Control D) None of the above, Special Revenue Funds do not record budgets

B) A debit to Estimated Revenues Control

Revenue bonds sold by a water utility fund, upon sale, would be recorded in an enterprise fund as: A) Other Financing Sources—Proceeds of Bonds. B) A liability. C) A direct addition to Net Assets. D) Nonoperating Revenues—Proceeds of Bonds.

B) A liability.

Which of the following statements is true regarding estimated closure costs for municipal solid waste landfills, assuming that it is operating as an enterprise fund? A) The estimated closure costs are expensed fully at the time of the estimate. B) A portion of those future estimated costs is to be charged as an expense and a liability recorded based on a units-of-production method. C) Modified accrual principles apply and the amount expensed is limited to amounts expected to be paid during the year and within 60 days of year end. D) None of the above is true.

B) A portion of those future estimated costs is to be charged as an expense and a liability recorded based on a units-of-production method.

Proprietary funds utilize what basis of accounting? A) Modified accrual. B) Accrual. C) Cash. D) Budgetary.

B) Accrual.

The terms used for classification of items on the Statement of Changes in Fiduciary Net Assets are: A) Revenues and Expenses. B) Additions and Deductions. C) Revenues and Expenditures. D) Contributions and Expenses.

B) Additions and Deductions.

Governmental funds do not include: A) Special revenue funds B) Agency funds C) Debt service funds D) The General fund

B) Agency funds

Pollution remediation costs: A) Are recorded as expenses if they will be paid from current resources. B) Are to be accrued as a liability in the basic financial statements. C) Are not an issue is the state or local government merely used (but did not operate) the polluted site. D) All of the above

B) Are to be accrued as a liability in the basic financial statements.

138. How would the $40,000 premium be accounted for? A) Amortized to interest expenditure in the debt service fund. B) As an other financing source in the debt service fund. C) Both (a) and (b) above. D) None of the above.

B) As an other financing source in the debt service fund.

When supplies ordered by a governmental unit are received at an actual price which is less than the estimated price on the purchase order, the Encumbrance Control account is: A) Credited for the actual price on the purchase invoice. B) Credited for the original estimated price for the supplies. C) Debited for the original estimated price for the supplies D) Debited for the actual price on the purchase invoice.

B) Credited for the original estimated price for the supplies.

Which of the following statements is not true regarding modified accrual accounting? A) Expenditures and fund liabilities are recorded when goods and services are received. B) Debt service expenditures for principal and interest are accrued. C) Expenditures for claims and judgments and landfill post-closure care costs of governmental funds should be recognized to the extent that the liabilities are going to be paid with available resources. D) Compensated absences and special termination benefits of governmental funds that will not be paid with available resources should be reported as liabilities in the government-wide statements.

B) Debt service expenditures for principal and interest are accrued.

Sales taxes, income taxes, and motor fuel taxes are examples of which class of nonexchange transactions? A) Imposed nonexchange transactions. B) Derived tax revenues. C) government-mandated nonexchange transactions. D) Voluntary nonexchange transactions.

B) Derived tax revenues.

GASB requires the ____________ method to report cash flows from operating activities in the Statement of Cash Flows. A) Indirect. B) Direct. C) Modified accrual D) Full accrual

B) Direct.

A private-purpose trust fund where the principal is not expendable, normally it is called a/an: A) Donation. B) Endowment. C) Gift. D) Transfer.

B) Endowment.

Governmental units use which fund type to account for services provided to the general public on a user-charge basis? A) General Fund. B) Enterprise fund. C) Permanent fund. D) Internal service fund.

B) Enterprise fund.

The two types of proprietary funds include: A) Enterprise funds and capital projects funds. B) Enterprise funds and internal service funds. C) Internal service funds and capital project funds. D) Agency funds and enterprise funds.

B) Enterprise funds and internal service funds.

Which of the following is *not* a fiduciary fund type? A) Agency. B) Expendable trust. C) Private-purpose trust. D) Investment trust.

B) Expendable trust.

Which of the following is true regarding modified accrual accounting?

B) Expenditures for services used by General Fund departments should be recognized when those services are received, regardless of whether or not resources are available in the fund

An internal service fund provided services to a General Fund department. At the time of billing, the debit entry in the General Fund would be: A) Other Financing Sources-Transfers Out. B) Expenditures. C) Operating Revenues-Charges for Services. D) Expenses.

B) Expenditures.

Investments in a private-purpose trust fund should generally be reported using: A) Cost Basis. B) Fair Market Value. C) Equity Method. D) None of the above.

B) Fair Market Value.

GASB Statement No. 52 requires endowments with investment real estate to report those assets at: A) Historical cost B) Fair market value C) Historical cost net of related debt D) Market value net of related debt

B) Fair market value

Which of the following is true regarding the financial statements of fiduciary funds? A) Fiduciary funds' financial statements include the Statement of Fiduciary Net Assets, the Statement of Changes in Fiduciary Net Assets, and the Statement of Fiduciary Cash Flows. B) Fiduciary funds are accounted for using the economic resources measurement focus and accrual basis of accounting. C) Both of the above. D) Neither of the above.

B) Fiduciary funds are accounted for using the economic resources measurement focus and accrual basis of accounting.

What is the effect on the General Fund's fund balance when capital assets are purchased during a year from General Fund revenues? A) Fund balance is increased. B) Fund balance is decreased. C) There is no effect, since one asset (cash) is replaced by another (capital asset). D) None of the above, capital assets must be purchased through a capital projects fund.

B) Fund balance is decreased

When delinquent taxes are written off: A) Fund balance will decrease B) Fund balance will be unaffected C) Expenditures will increase D) Both a and c.

B) Fund balance will be unaffected

Which of the following is not true regarding postemployment benefit plans other than pension plans? A) Most governments have operated on a pay-as-you-go basis, leaving such plans underfunded. B) GASB Statement No. 43, Financial Reporting for Postretirement Benefit Plans Other Than Pension Plans, requires that such plans be fully funded. C) Similar to pensions, governments are required to present two financial statements, the Statement of Plan Net Assets and the Statement of Changes in Plan Net Assets. D) Two schedules must be included as required supplementary information: the Schedule of Funding Progress and the Schedule of Employer Contributions.

B) GASB Statement No. 43, Financial Reporting for Postretirement Benefit Plans Other Than Pension Plans, requires that such plans be fully funded.

An example of an activity that would not normally be accounted for in an internal service fund would be: A) Motor pool. B) Garbage collection. C) Print shop. D) Central storeroom.

B) Garbage collection.

The ______ Fund accounts for all resources other than those required to be accounted for in other funds. A) Special revenue B) General C) Enterprise D) Agency

B) General

Which of the following fund types is present in every general-purpose government? A) Permanent B) General C) Special revenue D) Capital projects

B) General

Which Statement is presented using the Accrual Basis of Accounting? A) Governmental Fund Balance Sheet B) Government-Wide Statement of Activities C) Statement of Revenues, Expenditure and changes in Fund Balances D) A and C

B) Government-Wide Statement of Activities

Level "A" GAAP for The University of Virginia, a public institution, would be established by the: A) Financial Accounting Standards Board. B) Governmental Accounting Standards Board. C) American Institute of Certified Public Accountants. D) National Association of College and University Business Officers.

B) Governmental Accounting Standards Board.

Under which fund type would you debit expenditure when land is acquired? A) Proprietary. B) Governmental. C) Both of the above. D) None of the above.

B) Governmental.

Which of the following statements is not correct with respect to GASB Statement No. 53 which establishes reporting requirements for governments entering into derivative instruments? A) Derivatives include swaps, options, forward contracts and future contracts. B) If a derivative is effective in reducing a government's exposure to identifiable risks, the changes in the value of the derivative are reflected as investment gains or losses in the period that the value changes. C) If a hedge derivative is deemed ineffective it is classified as investment purpose. D) Statement No. 53 applies only to reporting at the government-wide level for derivatives held by governmental funds, not the fund-basis statements.

B) If a derivative is effective in reducing a government's exposure to identifiable risks, the changes in the value of the derivative are reflected as investment gains or losses in the period that the value changes.

68. Property Taxes, special assessments, and fines and forfeits are types of nonexchange transactions. These are examples of: A) Government-mandated nonexchange transactions. B) Imposed nonexchange transactions. C) Deferred tax revenues. D) Voluntary nonexchange transactions.

B) Imposed nonexchange transactions.

Revenue bonds sold by a water utility fund, upon sale, would be recorded: A) In an enterprise fund as "Other Financing Source." B) In an enterprise fund as a liability. C) In the general long-term debt accounts as a liability. D) In an enterprise fund as an addition to net assets.

B) In an enterprise fund as a liability.

Which of the following is not an objective of federal financial reporting? A) Budgetary Integrity B) Increase Taxpayer Awareness C) Stewardship D) Operating Performance

B) Increase Taxpayer Awareness

Regarding interfund transactions, which of the following is not correct? A) Transaction between funds are not assumed to be arm's length in nature B) Interfund transactions must be repaid. C) Failure to properly report interfund transactions will result in two funds being misstated. D) Interfund reimbursement is a payment by one fund to second fund to repay it for a purchase which belongs in the first fund.

B) Interfund transactions must be repaid.

Financial reports for state and local governments are well suited for all the following EXCEPT? A) Assessing financial condition and results of operations B) Measuring the effectiveness of government services such as police and fire protection C) Comparing actual results with the legally adopted budget D) Assisting in determining compliance with finance-related laws and regulations

B) Measuring the effectiveness of government services such as police and fire protection

An example of an activity that would not normally be accounted for in an enterprise fund would be: A) Swimming pool. B) Motor pool. C) Airport. D) Lottery.

B) Motor pool.

Which of the following would generally be included in the Statement of Net Assets of an Enterprise Fund? Reserve for Encumbrances Revenue Bonds Payable A) No No B) No Yes C) Yes No D) Yes Yes

B) No Yes

A government may have __________________ General Fund(s) in a given year? A) As many as needed B) Only one. C) The number will vary depending on the needs of the government. D) None of the above.

B) Only one.

The city is using an internal service fund to self-insure its risk financing activities. Claims accrued or paid by this fund are recorded as: A) Transfers out. B) Operating Expenses. C) Special Items. D) Nonoperating Expenses.

B) Operating Expenses.

An internal service fund provided services to a General Fund department. At the time of billing, the credit entry in the internal service fund would be: A) Other Financing Sources—Transfers In. B) Operating Revenues—Charges for Services. C) Other Financing Sources—Charges for Services. D) Due from General Fund.

B) Operating Revenues—Charges for Services.

The initial transfer of cash from the General Fund to establish a motor pool internal service fund would require the General Fund to credit Cash and debit: A) Investment in Internal Service Fund. B) Other Financing Uses-Transfers Out Control. C) Expenditures - Vehicle maintenance. D) Accounts Receivable-Internal Service Fund.

B) Other Financing Uses-Transfers Out Control.

Which of the following is *not"* a fiduciary fund? A) Agency B) Permanent C) Pension trust D) Private-purpose trust

B) Permanent

Which of the following funds is not a Fiduciary Fund? A) Private Purpose Trust Fund B) Permanent Fund C) Pension Fund D) Investment Trust Fund

B) Permanent Fund

Assume encumbrances do not expire at year-end. $16,000 was encumbered during the prior year for a computer and the actual cost of the computer in the current year is $14,000. How does this affect unassigned fund balance? A) Previous Year: increase $16,000; Current Year: decrease $14,000 B) Previous Year: decrease $16,000; Current Year: increase $2,000 C) Previous Year: no effect; Current Year: decrease $14,000 D) Previous Year: decrease $16,000; Current Year: decrease 14,000

B) Previous Year: decrease $16,000; Current Year: increase $2,000

A fund that is the result of an agreement between a contributor and a government that the principal and/or income of trust assets is for the benefit of individuals, organizations, or other governments is a(n): A) Agency fund B) Private-Purpose Trust Fund C) Investment Trust Fund D) Pension Trust Fund

B) Private-Purpose Trust Fund

A ___________ is used when a contributor and a government agree that the principal and/or income of trust assets are for the benefit of individuals. A) Investment Trust. B) Private-Purpose Trust. C) Pension Trust. D) Employee Benefit Trust.

B) Private-Purpose Trust.

Which of the following is true regarding fiduciary funds? A) Investment trust funds account for the internal portion of an investment pool, when a government is trustee for that pool. B) Private-purpose trust funds account for resources that are administered by a government for the benefit of others, such as a fund held to distribute resources to local not-for-profit charities. C) Both of the above. D) Neither of the above.

B) Private-purpose trust funds account for resources that are administered by a government for the benefit of others, such as a fund held to distribute resources to local not-for-profit charities.

Escheat property is best described as which of the following? A) Property possessed by a local government, but whose use is restricted by a higher government. B) Property a government obtains and holds for the rightful owner in the absence of claimants or heirs. C) Property gifted to the government by private citizens. D) Property the government obtains via eminent domain.

B) Property a government obtains and holds for the rightful owner in the absence of claimants or heirs.

Capitalized fixed assets are reported in which of the following fund-basis statements? A) Agency fund B) Proprietary fund C) Governmental fund D) All of the above

B) Proprietary fund

Which of the following is not an example of an Enterprise Fund? A) Electric and Water Utilities. B) Public Libraries. C) Sports Stadiums. D) All of the above are commonly reported as enterprise funds.

B) Public Libraries

The expenditures account equals the amount of supplies purchased during a period under which of the following inventory methods? A) Perpetual. B) Purchases. C) Periodic. D) Consumption.

B) Purchases.

Which of the following results in an expenditure? A) Place a purchase order. B) Receive goods previously ordered. C) Receive an invoice on goods previously received. D) Payment of an invoice previously received.

B) Receive goods previously ordered.

When recording property taxes, the estimated uncollectible amount of property taxes is: A) Recognized as an expenditure. B) Recognized as a reduction of revenue. C) Not recognized. D) None of the above.

B) Recognized as a reduction of revenue.

Debt service expenditures for interest are: A) Accrued. B) Recorded when due but may be accrued at year end if the debt service due date is less than 30 days after year end. C) Recorded when due but may be accrued at year end if the debt service due date is less than 60 days after year. D) Reported only in the government-wide statements.

B) Recorded when due but may be accrued at year end if the debt service due date is less than 30 days after year end.

The modified accrual basis is a distinct system of accounting that: A) Differs from accrual accounting only in its failure to recognize long term assets and liabilities. B) Records expenditures and fund balances. C) Records expenses and revenues the same as accrual accounting D) Is equivalent to cash basis accounting

B) Records expenditures and fund balances.

Which of the following items would typically not need an encumbrance? A) New Building B) Regular Salaries C) New Computer D) Office Equipment

B) Regular Salaries

The modified approach for infrastructure requires schedules and disclosures to be included in which part of the CAFR? A) Notes to the Financial Statements B) Required Supplementary Information C) Other Supplementary Information D) Proprietary Fund Statements

B) Required Supplementary Information

79. According to GASB Statement No. 54, spendable fund balances are displayed in which of the following categories? A) Restricted, Unrestricted, Assigned, Unassigned B) Restricted, Committed, Assigned, Unassigned C) Spendable, Assigned, Unassigned D) Restricted, Unrestricted

B) Restricted, Committed, Assigned, Unassigned

Which of the following is true regarding modified accrual accounting?

B) Revenues are recognized in the period in which they become available and measurable, and expenditures are recognized at the time a liability is incurred, except for principal and interest on long-term debt.

Which of the following General Fund accounts would be closed at year end? A) Due from State Government. B) Special Items - Proceeds from Sale of Land. C) Taxes Receivable - Delinquent. D) Deferred Inflows - Property Taxes.

B) Special Items - Proceeds from Sale of Land.

The purpose of an encumbrance is to prevent governments from: A) Overriding Unauthorized Contracts B) Spending in Excess of the Amounts Authorized C) Revising the Approved Budget D) Misappropriating Tax Revenues

B) Spending in Excess of the Amounts Authorized

Which financial statements are required for a proprietary fund? A) Income statement, Statement of Net Assets and Statement of Cash Flows B) Statement of Revenues, Expenses and Changes in Net Assets, Statement of Net Assets and Statement of Cash Flows C) Statement of Revenues, Expenses and Changes in Fund Balance and Statement of Net Assets, and Statement of Cash Flows D) Statement of Revenues, Expenses and Changes in Net Assets and Statement of Net Assets

B) Statement of Revenues, Expenses and Changes in Net Assets, Statement of Net Assets and Statement of Cash Flows

At what point would the General Fund account "Budgetary Fund Balance -- Reserve for Encumbrances" be debited? A) When an item is ordered by the government. B) When an authorized liability is incurred (the government receives the order). C) When cash is paid to the vendor. D) (A) or (B) depending on authorized appropriations.

B) When an authorized liability is incurred (the government receives the order).

Which of the following is not true regarding proprietary funds? A) Accrual accounting and the economic resource measurement focus are used. B) Statements required are the Balance Sheet, the Statement of Revenues, Expenditures, and Changes in Fund Balances, and the Statement of Cash Flows. C) Long Term debt is recorded directly in the accounts. D) Proprietary funds financial reports include the Statement of Net Assets, Statement of Revenues, Expenses, and Changes in Fund Net Assets, and Statement of Cash Flows.

B) Statements required are the Balance Sheet, the Statement of Revenues, Expenditures, and Changes in Fund Balances, and the Statement of Cash Flows.

Where is the actuarial liability shown in a Comprehensive Annual Financial Report for a pension trust fund? A) The Statement of Fiduciary Net Assets. B) The Schedule of Funding Progress (RSI). C) The General Fund Balance Sheet only. D) The Government-wide Statement of Net Assets.

B) The Schedule of Funding Progress (RSI).

Which of the following is not true regarding pension accounting and reporting? A) Local governments include only the pension plans for which it is trustee in pension trust funds Statement of Fiduciary Net Assets. B) The pension trust fund Statement of Fiduciary Net Assets reflects the accrued actuarial liability (the excess of the projected benefit obligation over the net assets available for benefits). C) Pension investments are recorded at fair (market) value. D) None of the above; all are true.

B) The pension trust fund Statement of Fiduciary Net Assets reflects the accrued actuarial liability (the excess of the projected benefit obligation over the net assets available for benefits).

Agency Funds are not included in the Government-wide Financial Statements because: A) These Funds use the modified accrual basis of accounting. B) The resources are not available to the reporting unit. C) The government doesn't keep track of these types of funds. D) These funds do not have a material effect on the financial statements.

B) The resources are not available to the reporting unit.

For landfills accounted for as enterprise funds, a portion of the estimated cost of closure of solid waste landfills should be charged as an expense and a liability of the landfill operation each year on a: A) Actual cost method. B) Units-of-production method. C) Estimated cost method. D) None of the above.

B) Units-of-production method.

Expenditures are generally recorded and fund liabilities are recognized

B) When goods and services are received, regardless of whether or not resources are available in the fund.

Which of the following is true regarding pension (and other employee benefit) trust funds? A) The actuarially computed Pension Benefit Obligation is reported in the Statement of Fiduciary Net Assets. B) While full accrual accounting is used, the terms "additions" and "deductions" are used in the Statement of Changes in Fiduciary Net Assets in lieu of "revenues" and "expenses." C) Both of the above. D) Neither of the above.

B) While full accrual accounting is used, the terms "additions" and "deductions" are used in the Statement of Changes in Fiduciary Net Assets in lieu of "revenues" and "expenses."

Revenues in governmental fund accounting... A) include taxes, fees, resources provided by other governments, and interfund transfers B) are recognized in the fiscal year they are available for expenditure C) are recognized when earned D) none of the above describes revenues in governmental accounting

B) are recognized in the fiscal year they are available for expenditure

When supplies ordered for use in an activity accounted for in the General Fund of a government are received at an actual price which is less than the estimated price on the purchase order, the Encumbrance Control account is: A) debited for the estimated price on the purchase order. B) credited for the estimated price on the purchase order. C) debited for the actual price for the supplies received. D) credited for the actual price of the supplies received.

B) credited for the estimated price on the purchase order.

Government-wide statements use the: A) current financial resources measurement focus and accrual basis of accounting. B) economic resources measurement focus and accrual basis of accounting. C) economic resources measurement focus and modified accrual basis of accounting. D) none of the above.

B) economic resources measurement focus and accrual basis of accounting

Under GASB Statement 34, capital assets: A) must be reported in government-wide statements but are not reported in any of the fund financial statements. B) must be reported in government-wide statements and in proprietary fund financial statements. C) are not to be reported in either government-wide or fund financial statements. D) are to be reported but not depreciated in government-wide and fund financial statements.

B) must be reported in government-wide statements and in proprietary fund financial statements.

A city government collects local option sales taxes legally restricted to pay for the construction of a new courthouse. Which fund should account for the receipt of the sales taxes? A)Special revenue fund B)Capital projects fund C)Private-purpose trust fund D)General fund

B)Capital projects fund

Which of the following is true regarding component units? A)Component units could include towns, school districts, counties, and municipalities. B)Component units may be reported discretely in the government-wide statements or may be blended as a fund in the fund financial statements (and thus also included in the government-wide financial statements). C)Both of the above. D)Neither of the above.

B)Component units may be reported discretely in the government-wide statements or may be blended as a fund in the fund financial statements (and thus also included in the government-wide financial statements).

Which of the following is true regarding the proprietary funds statements? A)Major enterprise and internal service funds are reported in separate columns; a column is presented for all non-major enterprise and internal service funds (combined), and a total column is presented. B)Financial statements include a Statement of Net Assets (or balance sheet), Statement of Revenues, Expenses, and Changes in Fund Net Assets, and Statement of Cash Flows. C)Both of the above. D)Neither of the above.

B)Financial statements include a Statement of Net Assets (or balance sheet), Statement of Revenues, Expenses, and Changes in Fund Net Assets, and Statement of Cash Flows.

An example of an expenditure classification by function would be: A)Current. B)Public safety. C)Police Department. D)Salaries.

B)Public safety.

Which of the following is true regarding the composition of the Comprehensive Annual Financial Report (CAFR)? A)The CAFR is required in order to be in conformity with Generally Accepted Accounting Principles (GAAP). B)The CAFR is to include both blended and discretely presented component units. C)The CAFR contains four major sections: introductory, financial, supplementary, and statistical. D)All of the above are true.

B)The CAFR is to include both blended and discretely presented component units.

Under GASB Statement 34, modified accrual accounting would be found in the: A)government-wide financial statements only. B)governmental fund financial statements only. C)governmental and fiduciary fund financial statements only. D)governmental, proprietary, and fiduciary fund financial statements but not in the government-wide financial statements

B)governmental fund financial statements only.

When a government acquires general fixed assets under a capital lease agreement, the asset should be recorded in the General Fund's financial statements:

B. As an expenditure when payments are made

When taxes and / or special assessments are levied specifically for payment of interest and principal on long -term debt, those taxes are recognized:

B. As revenues in the debt service fund

When a payment is due to a contractor from capital projects fund resources, the debit would be to:

B. Capital Expenditures

When a governmental unit is primarily or secondarily liable for the debt, debt proceeds for special assessment obligations levied for the construction phase of a capital improvement project should be reported in which of the following fund types?

B. Capital Projects

With respect to Capital Projects Funds, which of the following is correct?

B. Capital assets appear in both the government-wide and the fund basis financial statements.

Governmental fund statements are prepared using: A. Economic resources measurement focus and modified accrual basis of accounting. B. Current financial resources measurement focus and modified accrual basis of accounting. C. Economic resources measurement focus and accrual basis of accounting. D. Current financial resources measurement focus and accrual basis of accounting.

B. Current financial resources measurement focus and modified accrual basis of accounting.

Capital project funds:

B. Exist only for the duration of the project for which they are created.

Which of the following is not one of the objectives of Federal Financial Reporting, as outlined in SFFAC 1, which was issued by the FASAB? A. Financial reporting should enable evaluation of the service efforts, costs and accomplishments of the reporting entity. B. Financial reporting should enable evaluation of the success rate of programs C. Financial reporting should reveal whether financial systems and controls are adequate D. Financial reporting should demonstrate accountability regarding raising and expending money according to budgetary process and laws and regulations

B. Financial reporting should enable evaluation of the success rate of programs

Which of the following use current financial resources measurement focus? A. Fiduciary fund statements B. Governmental fund statements C. Proprietary fund statements D. Internal Service fund statements

B. Governmental fund statements

If taxes and/or special assessments are levied by the General Fund, and then are subsequently transferred to the debt service fund, they are:

B. Included in the revenues of the General Fund and are also reported by the General Fund as transferred out to the debt service fund.

Budgets are typically recorded for which of the following:

B. Special Revenue Funds

With respect to Government-wide financial statements, which of the following is not required by GASB Statement 34? A. Statement of Net Assets B. Statement of Cash Flow C. Statement of Activities D. All of the above are required by GASB Statement 34

B. Statement of Cash Flow

The GASB sets accounting standards for all of the following except: A. State and local governments B. Nongovernmental not-for-profit hospital C, Component units owned or controlled by governments D. Governmentally related not-for-profit universities

B. Nongovernmental not-for-profit hospital

Cash provided by the General Fund for a capital project would be recorded in a capital projects fund as a (an):

B. Other Financing Sources

Capital project funds record the proceeds of debt issued as:

B. Other Financing Sources.

Which of the following is NOT true regarding permanent funds?

B. Permanent Funds are appropriate when a gift must be invested and the proceeds used to benefit individuals or organizations.

A non expendable trust which benefits a government or its citizenry and which stipulates that earning only ( not principal) may be used for its provided purpose should be reported in a:

B. Permanent fund

The FASB has the authority to establish accounting and financial reporting standards for: A. Federal government B. Private not-for-profits C. Public not-for-profits D. State and local governments

B. Private not-for-profits

Which of the following funds require a Statement of Cash Flows? A. Governmental funds B. Proprietary funds C. Fiduciary Funds D. Governmental and Fiduciary Funds

B. Proprietary funds

The following description provides the best definition for which fund: Accounts for resources that are legally restricted and both earnings and principal may be used to support government programs.

B. Special Revenue

Siler CIty receives a trust donation for the purpose of maintaining flower beds in city parks, but the donor does not specify that the principal must be maintained. This type of trust would be most appropriately accounted for in a:

B. Special Revenue Fund.

Which of the following is true with respect to the General Fund A. The General Fund is considered to be a major fund if the combined total of assets, liabilities, revenues and expenses exceeds 10% of the total of all governmental funds B. The General Fund is always considered to be a major fund when preparing fund basis financial statements. C. The General Fund is considered to be a major fund when preparing fund basis financial statements if it bears a financial benefit or burden to the primary government. D. The General Fund is not reported as part of the CAFR.

B. The General Fund is always considered to be a major fund when preparing fund basis financial statements.

Which of the following projects would normally be accounted for in a capital projects fund?

B. The construction of a parking garage operated as an enterprise fund.

Which of the following lease criteria would NOT qualify a lease as a capital lease?

B. The lease contains an option to purchase the leased property at its fair market value.

Premiums generated from the issuance of bonds for capital project fund are generally:

B. Transferred to the debt service fund

A local government was awarded a federal grant in the amount of $1,200,000 to provide for a summer employment program for young people. The grant was a reimbursement grant and was awarded on April 30, 2012. The local government expended the resources as follows: June, 2012, $440,000; July 2012, $400,000; August, 2012, $360,000. The federal government provided the funds the following months. The local government would recognize revenues for the fiscal year ended June 30, 2012 in which amount?

C) $ 440,000

The Township of Thomasville's General Fund has the following net resources at year end: • $77,000 of prepaid insurance • $375,000 rainy day fund approved by the township governing board with specific conditions for its use • $2,500 of supplies inventory • $61,000 state grant for snow removal • $150,000 contractual obligations for the purchase of equipment • $200,000 to be used to fund government operations in the future • Outstanding encumbrance of $80,000 for the purchase of furniture & fixtures 60. What would be the total Nonspendable fund balance? A) $ 2,500 B) $ 77,000 C) $ 79,500 D) $159,500

C) $ 79,500

The Fire Department of a certain city received an appropriation in the amount of $13,000,000 for the fiscal year ended December 31, 2014. During the month ended January 31, 2014, the following transactions occurred: (a) purchase orders were issued in the amount of $480,000; (b) purchase orders, related to (a) above, were filled in the amount of $380,000; the related invoice amount was $368,000; invoices were paid in the amount of $350,000; (c) salaries were accrued and paid in the amount of $575,000; (d) the appropriation was increased in amount of $100,000. The amount available for the Fire Department as of January 31, 2014 would be: A) $11,787,000. B) $12,077,000. C) $12,057,000. D) $12,137,000.

C) $12,057,000.

James McHughes gave the following to the City of Carnesville in order to establish a private-purpose trust: o Land - cost, $500,000; fair market value as of the date of the gift, $400,000. o Securities - cost, $1,600,000; fair market value as of the date of the gift, $1,800,000. The amount to be recorded as additions for gifts by the private-purpose trust fund would be: A) $2,000,000. B) $2,100,000. C) $2,200,000. D) $2,300,000.

C) $2,200,000.

140. What would be the amount of expenditures recorded by the debt service fund for the fiscal year ended December 31, 2016? A) $440,000. B) $324,000. C) $320,000. D) $120,000.

C) $320,000.

The City of Thomasville had the following debt outstanding: General obligation bonds to be paid from a debt service fund $3,900,000 General obligation bonds to be paid from utility revenues $3,100,000 Revenue bonds to be paid from utility revenues $2,400,000 The amount that should be shown as debt in the utility (enterprise) fund would be: A) $2,400,000. B) $3,100,000. C) $5,500,000. D) $9,400.000

C) $5,500,000.

The Township of Thomasville's General Fund has the following net resources at year end: • $77,000 of prepaid insurance • $375,000 rainy day fund approved by the township governing board with specific conditions for its use • $2,500 of supplies inventory • $61,000 state grant for snow removal • $150,000 contractual obligations for the purchase of equipment • $200,000 to be used to fund government operations in the future • Outstanding encumbrance of $80,000 for the purchase of furniture & fixtures What would be the total Committed fund balance? A) $375,000 B) $290,000 C) $525,000 D) $455,000

C) $525,000

The Police Department of a given city received an appropriation in the amount of $8,000,000 for the fiscal year ended June 30, 2014. During the month ended July 31, 2013, the following transactions occurred: (a) purchase orders were issued in the amount of $500,000; (b) purchase orders, related to (a) above, were filled in the amount of $490,000; the related invoice amount was $495,000; invoices were paid in the amount of $450,000; (c) salaries were accrued and paid in the amount of $475,000. The balance available for the police department as of July 31, 2013 is: A) $7,065,000. B) $7,025,000. C) $7,020,000. D) $6,575,000.

C) $7,020,000.

The statistical section typically presents ____ years of information in each table or schedule. A) 3 B) 5 C) 10 D) 12

C) 10

GASB provides which method(s) for including component unit financial information with that of the primary government? A) Discrete Presentation B) Blending C) A and B D) None of the above

C) A and B

The journal entry to write off an uncollectible property tax receivable for a municipality would include: A) A credit to Estimated Uncollectible Current Taxes B) A debit to Bad Debts Expense C) A credit to Taxes Receivable - Current D) B and C would both be included in the journal entry

C) A credit to Taxes Receivable - Current

Which of the following is true regarding the Statement of Cash Flows required by the GASB for proprietary funds? A) Either the direct or indirect method may be used. B) Interest payments would be recorded as investing activities. C) Acquisition of capital assets with bond proceeds would be a decrease in cash provided by capital and related financing activities. D) Interest received on investments is reported as cash flows from operating activities.

C) Acquisition of capital assets with bond proceeds would be a decrease in cash provided by capital and related financing activities.

Fund equity will not be found in... A) Investment trust funds. B) Pension trust funds. C) Agency funds. D) Private-purpose trust funds.

C) Agency funds.

Revenue bonds A) Carry less risk than general obligation bonds. B) Are no longer in existence. C) Are intended to be payable from the revenues of an enterprise. D) Are backed by the full faith and credit of the government in addition to enterprise revenues.

C) Are intended to be payable from the revenues of an enterprise.

Interfund transfers occur between individual funds in the same government system without the requirement of repayment. If the General Fund transferred money to the Debt Service Fund, how would the Debt Service Fund account for the transaction? A) As non-operating revenue. B) As a liability on the balance sheet. C) As a source of funds on the activities statement. D) As a reduction in net liabilities.

C) As a source of funds on the activities statement.

137. How would the government account for the transfer of the unused bond proceeds? A) As a revenue in the debt service fund and as an expenditure in the capital projects fund. B) As an other financing source in the capital projects fund and as an other financing use in the debt service fund. C) As an other financing source in the debt service fund and as an other financing use in the capital projects fund. D) As a special item in both the debt service and capital project funds.

C) As an other financing source in the debt service fund and as an other financing use in the capital projects fund.

Which of the following is true regarding modified accrual accounting? A) Revenues are recognized when measurable and available to finance expenditures of the current period. B) Expenditures are generally recognized as the related goods or services are received. C) Both (a) and (b) above. D) None of the above.

C) Both (a) and (b) above.

When accounting for other postemployment benefits, the OPEB obligation for employees of governmental activities is: A) Reported in the government-wide statements, but not the fund basis statements. B) The cumulative difference between the amount funded and the annual required contribution. C) Both A and B. D) Neither A nor B.

C) Both A and B.

Which of the following is true regarding fiduciary funds? A) Fiduciary funds are included in the fund basis statements but not in the government-wide. B) Fiduciary funds are reported by fund type, not as major funds. C) Both of the above. D) Neither of the above.

C) Both of the above

Which of the following is true regarding agency funds? A) Agency funds report assets and liabilities, but not net assets, revenues or expenses. B) Agency funds are often used to account for property tax collections by county governments for other governments. C) Both of the above. D) Neither of the above.

C) Both of the above.

Which of the following is true regarding fiduciary funds? A) Fiduciary funds are not included in the government-wide financial statements. B) Fiduciary funds include agency, pension (and other employee benefit) trust, private-purpose trust, and investment trust funds. C) Both of the above. D) Neither of the above.

C) Both of the above.

Which of the following is true regarding fiduciary funds? A) When a government is trustee for a defined benefit pension plan, two RSI schedules are required: a schedule of funding progress and a schedule of employer contributions. B) Escheat property proceeds must be reported in either a private-purpose trust fund or the fund to which the property ultimately escheats. C) Both of the above. D) Neither of the above.

C) Both of the above.

Which of the following is true regarding pension accounting and reporting for state and local governments? A) When the General Fund makes a contribution to a pension trust fund, the account "Expenditures" is debited. B) The Statement of Changes in Fiduciary Net Assets reports "additions" and "deductions," computed on the accrual basis. C) Both of the above. D) Neither of the above

C) Both of the above.

Which of the following is true regarding the recording of long term debt in an enterprise fund? A) When revenue bonds are sold at par, Cash is debited and Bonds Payable is credited. B) The bonds would be reported in both an enterprise fund and in the government-wide statements. C) Both of the above. D) Neither of the above.

C) Both of the above.

Which of the following is true regarding the reporting of investments by state and local governmental units? A) Investments, for which a determinable fair value can be obtained, are to be reported at fair value. B) Realized and unrealized gains and losses are to be combined in the relevant operating statement (for example, the Statement of Changes in Fiduciary Net Assets). C) Both of the above. D) Neither of the above.

C) Both of the above.

Which of the following statements about agency funds is true? A) An agency relationship that usually results in the creation of an agency fund is the collection of taxes and other revenues by an official of one governmental unit for other governmental units. B) GASB mandates that governments report special assessment resources in an agency fund only if the reporting government has no obligation to assume debt service on special assessment debt in the event property owners' default but merely perform the functions of billing and collecting the special assessments and paying interest and principal on the special assessment debt. C) Both of the above. D) Neither of the above.

C) Both of the above.

Which of the following accounts in the General Fund is credited when a purchase order is approved? A)Appropriations Control. B)Encumbrances Control. C)Budgetary Fund Balance -- Reserve for Encumbrances. D)Vouchers Payable.

C) Budgetary Fund Balance -- Reserve for Encumbrances.

With respect to budgetary reporting by governments, which of the following is not true? A) While GASB standards guide the format of the comparison, it does not require governments to maintain budgetary accounts. B) The basis of accounting used to report the "Actual" column in the budgetary comparison schedule is prepared according to legal requirements for budget preparation; even if it is a departure from GASB standards. C) Budgetary accounts are required to appear in the general purpose financial statements. D) GASB standards require governments to present a comparison of budgeted and actual results for the General Fund and special revenue funds with legally adopted budgets.

C) Budgetary accounts are required to appear in the general purpose financial statements.

74. Impact fees imposed on commercial developers by an enterprise fund and not associated with specific projects or improvements are recorded as: A) Transfers from the enterprise fund to a capital projects fund. B) Operating revenues to the enterprise fund. C) Capital contributions to the enterprise fund. D) Unearned revenue to the enterprise fund.

C) Capital contributions to the enterprise fund.

Which of the following is not correct regarding the GASB Cash Flow Statement? A) The reconciliation of income and cash flows from operations starts with operating income. B) Fixed Assets acquisitions are reported in the Capital Related Financing Section. C) Cash received from Interest and Dividend Income is recorded in the Operating Section. D) All of the above are correct.

C) Cash received from Interest and Dividend Income is recorded in the Operating Section.

Estimated Revenues are $4,600,000, and Appropriations are $4,000,000, the journal entry for the budget includes a: A) Debit to Budgetary Fund balance for 600,000 B) Debit to Encumbrances for 600,000 C) Credit to Budgetary Fund balance for 600,000 D) Credit to Budgetary Fund Balance -- Reserve for Encumbrances for 600,000

C) Credit to Budgetary Fund balance for 600,000

The fund basis statements for governmental funds are presented using the A) Economic Resources Measurement focus and the Accrual Basis of Accounting B) Current Financial Resources Measurement focus and the Accrual Basis of Accounting C) Current Financial Resources Measurement focus and the Modified Accrual Basis of accounting D) Economic Resources Measurement focus and the Modified Accrual Basis of Accounting

C) Current Financial Resources Measurement focus and the Modified Accrual Basis of accounting

An employee enrolls in a pension plan that will pay out 3% of the employee's average salary for the last 3 years for each year of service that the employee worked. What type of pension plan does the employee have? A) Defined Contribution Plan. B) Normal Payout Plan. C) Defined Benefit Plan. D) None of the above.

C) Defined Benefit Plan.

Interfund Transfers are flows of cash or other assets that: A) Require repayment. B) Are an exchange between funds of equal value. C) Do not require repayment. D) Are taxable.

C) Do not require repayment.

Which account would be debited when the City of Corfu issued purchase orders for materials and supplies? A) Budgetary Fund Balance -- Reserve for Encumbrance. B) Expenditures. C) Encumbrances Control. D) Materials and supplies expense.

C) Encumbrances Control.

Which of the following would not be true regarding internal service funds? A) Internal service funds use the economic resources measurement focus and accrual basis of accounting. B) Examples of internal service funds would include self-insurance funds, motor pool funds, and print shop funds. C) In the government-wide financial statements, internal service funds are most commonly reported in the business-type activities column. D) None of the above; all are true.

C) In the government-wide financial statements, internal service funds are most commonly reported in the business-type activities column.

Which of the following statements is not correct? A)Transactions between funds of the same government may not be assumed to be arms' length in nature. B)Most interfund transactions are eliminated in the government-wide statements C)Interfund reimbursements are classified as other financing sources or uses D)Interfund transfers are classified as other financing sources or uses

C) Interfund reimbursements are classified as other financing sources or uses

Which of the following is true regarding internal service funds? A) Internal service funds provide services primarily to external users on a user charge basis. B) Internal service funds normally record the annual budget in the accounts. C) Internal service funds use full accrual accounting. D) Internal service funds' capital assets are not accounted for in the accounts.

C) Internal service funds use full accrual accounting.

Which of the following fund types does not use modified accrual accounting? A) Debt Service B) Special Revenue C) Investment Trust D) Permanent

C) Investment Trust

A fund that exists when the government is the sponsor of a multigovernment investment pool and accounts for the external portion of the trust assets is a(n): A) Agency fund B) Private-Purpose Trust Fund C) Investment Trust Fund D) Pension Trust Fund

C) Investment Trust Fund

Which of the following is true regarding the final appropriations budget? A) Can be changed at any time by the accounting department B) Can be exceeded by actual appropriations C) Is legal and binding D) None of the above

C) Is legal and binding

In an agency fund, assets are equal to: A) Liabilities + Net Assets B) Net Assets C) Liabilities D) There are no assets in an Agency Fund

C) Liabilities

Governmental fund financial statements are to be prepared on the A) Accrual basis of accounting. B) Cash basis of accounting. C) Modified accrual basis of accounting. D) Tax basis of accounting.

C) Modified accrual basis of accounting.

Under the modified accrual basis of accounting, GASB standards for property tax revenue recognition provide that revenue should not be recognized for property taxes collected

C) More than 60 days after fiscal year end

Which of the following occurs when uncollectible delinquent taxes are written off? (Ignore interest and penalties) A) Delinquent Tax Receivable is increased. B) Accounts receivable goes down and the Estimated Uncollectible Taxes Account is increased. C) Net receivables remain unchanged. D) Uncollectible Delinquent Tax Expense is increased.

C) Net receivables remain unchanged.

What is the appropriate accounting for the signing of a contract to build a new water treatment facility by an Enterprise Fund? A) Debit Encumbrances. B) Credit Budgetary Fund Balance - Reserve for encumbrances. C) No entry is required. D) Both A & B.

C) No entry is required.

When would a special revenue fund be deemed to have satisfied the eligibility requirement of a reimbursement-type federal grant, under GASB Statement 33? A) Only after work is completely finished for the project. B) When a plan for use of the funds has been developed and approved. C) Only as qualified expenditures are incurred. D) When the grant is first approved by the granting agency.

C) Only as qualified expenditures are incurred.

Pension trust fund financial statements would be found in the Comprehensive Annual Financial Report of a governmental reporting entity: A) Only if the governmental unit desires to disclose the status of employee retirement plans to its employees. B) In all cases when the governmental unit has an obligation to provide funding for employee retirement. C) Only if the governmental unit is the trustee for the public employee retirement plan. D) Whenever governmental employees have a retirement plan, whether or not the governmental unit is obligated to provide funding.

C) Only if the governmental unit is the trustee for the public employee retirement plan.

A transfer should be reported in the operating statement of the General Fund as a(an): A) Revenue or Expenditure. B) Due from or Due to Other Funds. C) Other Financing Source or Use. D) None of the above, it is a direct increase or decrease to fund balance.

C) Other Financing Source or Use.

Which of the following is false regarding the cash flow statements of a proprietary fund? A) A reconciliation is required between the Statement of Revenues, Expenses, and Changes in Fund Net Assets and the cash flows from operating activities section of the Cash Flow Statement. B) Interest payments are reported as increases in cash flows from either capital and related financing or noncapital financing activities, whichever is appropriate. C) Purchases of equipment would be reported in the investing section. D) None of the above. - these are all true.

C) Purchases of equipment would be reported in the investing section.

What would be the effect on the General Fund's Fund Balance at the end of the current fiscal year of recording the purchase of a $5,000 new computer with a five year estimated useful life and zero salvage? A)Have no effect on the General Fund's Fund Balance. B)Reduce the General Fund's Fund Balance $1,000. C)Reduce the General Fund's Fund Balance $5,000. D)Reduce the General Fund's Fund Balance $6,000.

C) Reduce the General Fund's Fund Balance $5,000.

Which of the following statements is not true with respect to GASB Statement No. 54 regarding restricted fund balances? A) Restricted funds are those that are subject to constraints imposed by external parties or by law. B) Restricted fund balance can result from legally enforceable requirements that resources be used only for specific purpose. C) Restricted funds are the result of a self-imposed constraint by a government upon itself. D) All of the above are correct

C) Restricted funds are the result of a self-imposed constraint by a government upon itself.

The four classes of nonexchange transactions include all of the following except: A) Imposed nonexchange revenues. B) Derived tax revenues. C) Sales of Services. D) Voluntary nonexchange transactions.

C) Sales of Services.

69. Which of the following would not be a nonexchange transaction for a state government? A) Fines and forfeits. B) Property taxes. C) Sales of lottery tickets. D) Income taxes.

C) Sales of lottery tickets.

Which of the following is not likely to be recorded in a special revenue fund? A) Phone fees restricted to supporting the emergency 911 access system. B) Hotel taxes restricted to promoting tourism. C) Sales taxes restricted to courthouse additions. D) State motor fuel tax restricted to road maintenance.

C) Sales taxes restricted to courthouse additions

Which of the following is not a required supplementary information schedule for pension pensions? A) Schedule of funding progress B) Schedule of employer contributions C) Schedule of employee contributions D) None of the above, all of these are required.

C) Schedule of employee contributions

Which of the following is true regarding the use of Special Revenue Funds? A)Special Revenue Funds may be used whenever a government wishes to segregate income for specific purposes. B)Special Revenue Funds are appropriate if the sole source of resources are assigned funds C)Special Revenue Funds may only be used when a substantial portion of the resources are provided by restricted or committed revenue sources. D)Once a Special Revenue Fund is established, it will continue to be a Special Revenue Fund until all resources are exhausted.

C) Special Revenue Funds may only be used when a substantial portion of the resources are provided by restricted or committed revenue sources

Which of the following is true regarding the use of Special Revenue Funds? A) Special Revenue Funds may be used whenever a government wishes to segregate income for specific purposes. B) Special Revenue Funds are appropriate if the sole source of resources are assigned funds C) Special Revenue Funds may only be used when a substantial portion of the resources are provided by restricted or committed revenue sources. D) Once a Special Revenue Fund is established, it will continue to be a Special Revenue Fund until all resources are exhausted.

C) Special Revenue Funds may only be used when a substantial portion of the resources are provided by restricted or committed revenue sources.

Which of the following is not true with respect to Special revenue funds? A)Resources appropriately reported within proprietary or fiduciary funds are excluded from special revenue funds. B)A special revenue funds would be used to account for taxes which are required by state law to be used by the government to support specific activities. C)Special revenue funds may be used for any designated purpose to segregate revenues and ensure compliance with restrictions. D)None of the above

C) Special revenue funds may be used for any designated purpose to segregate revenues and ensure compliance with restrictions

Which of the following statements is not required for pension trust funds? A) Statement of Fiduciary Net Assets. B) Statement of Changes in Fiduciary Net Assets. C) Statement of Fiduciary Cash Flows. D) None of the above; all three statements should be prepared for pension-trust funds.

C) Statement of Fiduciary Cash Flows.

All of the following are true about proprietary funds EXCEPT: A) They include Internal Service Funds and Enterprise Funds. B) The economic resources measurement focus is used. C) They include the Enterprise and Special Revenue Funds. D) Accrual accounting is used as the basis of accounting.

C) They include the Enterprise and Special Revenue Funds.

When payrolls and other liabilities are incurred and must be paid before substantial amounts of cash will be collected, what type of short-term note is desirable and secured by a government's power to tax?

C) Tax Anticipation Note Payable

Churchville County is trustee for a multi-government investment pool and has established an investment trust fund. Included in the investment trust fund, for management purposes, are investments in the amount of $15 million from the County's General Fund, $3 million from the County's special revenue funds, and $112 million from other governments. Which of the following would be true? A) The County would report $18 million in an investment trust fund. B) The County would report the entire $130 million in an investment trust fund. C) The County would report the $112 million in an investment trust fund, the $15 million in its General Fund, and the $3 million in special revenue funds. D) The County would report the $112 million in an investment trust fund and the$18 million in a permanent fund.

C) The County would report the $112 million in an investment trust fund, the $15 million in its General Fund, and the $3 million in special revenue funds.

The City of Brownsville had a balance in the Budgetary Fund Balance -- Reserve for Encumbrances account at the end of 2014 in the amount of $90,000. During 2015, all purchase orders related to the $90,000 were filled, and the invoice amount was $91,200. Which of the following would be true regarding the Statement of Revenues, Expenditures, and Changes in Fund Balances for 2015? (assume encumbrances do not lapse) A) The amount shown for Expenditures would include only the $1,200. B) The amount shown for Expenditures would include only the $90,000. C) The amount shown for Expenditures would include the $91,200. D) The amount shown for Expenditures would not include items related to orders placed in 2014.

C) The amount shown for Expenditures would include the $91,200

The "external portion" of an investment trust fund relates to... A) The portion of the investment that legally belongs to the sponsoring government. B) The portion of the investment that is allocated among the sponsoring government's funds. C) The portion of the investment that belongs to other participating governments. D) "External portion" relates only to agency funds, not investment trust funds.

C) The portion of the investment that belongs to other participating governments.

GASB Concepts Statement No. 3, Communication Methods in General Purpose External Financial Reports that Contain Basic Financial Statements, states that A) notes to the financial statements may include management's objective explanations. B) required supplementary information must be objective and may not include predictions or subjective assessments. C) disclosure in the notes is not an adequate substitute for recognition in the financial statements. D) All of the above are correct.

C) disclosure in the notes is not an adequate substitute for recognition in the financial statements.

Fiduciary funds are to use the: A) economic resources measurement focus and modified accrual basis of accounting. B) current financial resources measurement focus and accrual basis of accounting. C) economic resources measurement focus and accrual basis of accounting. D) none of the above, the fiduciary funds have no revenues.

C) economic resources measurement focus and accrual basis of accounting.

Under GASB Statement 34, accrual accounting is used for: A) government-wide financial statements only. B) government-wide financial statements and proprietary fund financial statements only. C) government-wide, proprietary fund, and fiduciary fund financial statements only. D) government-wide and all fund financial statements

C) government-wide, proprietary fund, and fiduciary fund financial statements only.

Under the modified accrual basis of accounting, revenues should be recognized when they are: A) authorized by the budget ordinance. B) realizable and earned. C) measurable and available to finance expenditures of the current period. D) realized through collection.

C) measurable and available to finance expenditures of the current period.

A statewide pension plan exists for all local governments in a certain state. The provisions of the plan indicate that each qualifying retiree receive 2% multiplied by the number of years active employment multiplied by the average salary for the past four years of service. The government calculates the actuarial liability on a statewide basis, not by individual government. The plan would be known as a (an): A) multiple-employer, defined benefit, agency plan. B) multiple-employer, defined contribution plan. C) multiple-employer, defined benefit, cost-sharing plan. D) Single employer plan.

C) multiple-employer, defined benefit, cost-sharing plan

GASB standards ____________ that each governmental reporting entity display ___________ General Fund in its general-purpose financial statements. A) recommend, one or more B) recommend, only one C) require, only one D) require, one or more

C) require, only one

In addition to a Statement of Net Assets, Proprietary Fund level financial statements include which of the following? A) Statement of revenues, expenses, and changes in fund net assets. B) Statement of cash flows. C)Both of the above. D)Neither of the above.

C)Both of the above.

Which of the following is true regarding the Comprehensive Annual Financial Report (CAFR)? A)The CAFR has three main sections: introductory, financial, and statistical. B)Required Supplementary Information includes a Budgetary Comparison Schedule for the General Fund and all major special revenue funds that have a legally adopted annual budget (unless a statement is prepared). C)Both of the above. D)Neither of the above.

C)Both of the above.

Which of the following is true regarding the governmental fund financial statements? A)The governmental fund financial statements include the Balance Sheet and the Statement of Revenues, Expenditures, and Changes in Fund Balances. B)The governmental fund financial statements are prepared using the current financial resources measurement focus and the modified accrual basis of accounting. C)Both of the above. D)Neither of the above.

C)Both of the above.

Which of the following is true regarding the governmental fund statements? A)The Governmental Funds Balance Sheet does not reflect capital assets or long-term debt. B)The Governmental Funds Statement of Revenues, Expenditures, and Changes in Fund Balances reports columns for major funds, non-major funds (in total), and total. C)Both of the above. D)Neither of the above.

C)Both of the above.

Which of the following is true regarding the proprietary fund financial statements? A)The proprietary fund financial statements include the Statement of Net Assets, the Statement of Revenues, Expenses, and Changes in Fund Net Assets, and the Statement of Cash Flows. B)The proprietary fund financial statements are prepared using the economic resources measurement focus and the accrual basis of accounting. C)Both of the above. D)Neither of the above.

C)Both of the above.

Which of the following is true regarding the reporting of Budget-Actual Comparisons? A)A Budget-Actual Comparison Schedule is required for the General Fund and all major special revenue funds that have a legally adopted annual budget. B)A Budget-Actual Comparison Statement may be prepared in lieu of the Schedule. C)Both of the above. D)Neither of the above.

C)Both of the above.

Which of the following is true regarding the reporting of major funds in the governmental funds financial statements? A)The General Fund is always considered a major fund. B)A fund is considered major if its revenues, expenditures, assets, or liabilities exceed 10% of the total governmental fund categories and if the same element (revenues, expenditures, assets, or liabilities) exceeds 5% of the governmental and enterprise funds combined. C)Both of the above. D)Neither of the above.

C)Both of the above.

Which of the following accounts of a governmental unit's General fund is debited when supplies previously ordered are received? A)Encumbrances Control. B)Reserve for Supplies. C)Budgetary Fund Balance -- Reserve for Encumbrances. D)Appropriations.

C)Budgetary Fund Balance -- Reserve for Encumbrances.

Which of the following would be accounted for as a permanent fund:

C. A gift of $75,000 to a city to be invested permanently, with the proceeds to be used to buy books for the city library.

In February, X City ordered $550,000 of supplies. On March 17th it received all of the ordered supplies and an invoice of $551,000. Which of the following would be recorded upon receipt of the supplies and invoice? A) Debit Encumbrances Control for $550,000. B) Debit Encumbrances Control for $551,000. C) Debit Budgetary Fund Balance -- Reserve for Encumbrances for $550,000. D) Debit Budgetary Fund Balance -- Reserve for Encumbrances for $551,000. Answer: C

C)Debit Budgetary Fund Balance -- Reserve for Encumbrances for $550,000.

The General Fund of the City of Bangor purchased water from its Water Utility Fund in the amount of $20,000. The General Fund would debit: A)Water Expense. B)Other Financing Uses-Transfers Out. C)Expenditures Control. D)None of the above; no entries would be made.

C)Expenditures Control.

Notre Dame University, a private institution, has level "A" GAAP established by the: A)American Institute of Certified Public Accountants. B)Governmental Accounting Standards Board. C)Financial Accounting Standards Board. D)National Association of College and University Business Officers

C)Financial Accounting Standards Board.

Which of the following is true regarding Management's Discussion and Analysis (MD&A)? A)MD&A is not considered part of Required Supplementary Information (RSI). B)MD&A takes the place of footnotes. C)MD&A should present an analysis of significant variations between the original and final budget. D)All of the above are true.

C)MD&A should present an analysis of significant variations between the original and final budget.

Which of the following is not considered Required Supplementary Information (RSI)? A)Management's Discussion and Analysis. B)Budgetary Comparison Schedule. C)Notes to the financial statements. D)None of the above; all are considered RSI.

C)Notes to the financial statements.

Which of the following is true regarding the proprietary fund financial statements? A)The Statement of Net Assets (or Balance Sheet) reflects equity as contributed equity and retained earnings. B)Normally, a reconciliation is required between the proprietary fund financial statements and the business-type activities column in the government-wide financial statements. C)Statements include the Statement of Net Assets, Statement of Revenues, Expenses, and Changes in Fund Net Assets, and Statement of Cash Flows. D)The Statement of Cash Flows may be prepared using either the direct or indirect methods.

C)Statements include the Statement of Net Assets, Statement of Revenues, Expenses, and Changes in Fund Net Assets, and Statement of Cash Flows.

Which of the following is true regarding the government-wide Statement of Activities? A)The government-wide Statement of Activities may reflect expenses either by function (general government, public safety, etc.) or by object or natural classification (salaries, supplies, etc.). B)The government-wide Statement of Activities is prepared using the modified accrual basis of accounting for governmental activities and using the accrual basis of accounting for business-type activities. C)The government-wide Statement of Activities reflects taxes as general revenues. D)Indirect program expenses may not be allocated to the other functional areas.

C)The government-wide Statement of Activities reflects taxes as general revenues.

Where in the CAFR would one find the long-term liability for revenue bonds (paid from the revenues of an enterprise fund)? A)The proprietary funds Statement of Net Assets only. B)The government-wide Statement of Net Assets only. C)The government-wide Statement of Net Assets and the proprietary funds Statement of Net Assets. D)The government-wide Statement of Net Assets and the RSI Schedule of Bonds Payable.

C)The government-wide Statement of Net Assets and the proprietary funds Statement of Net Assets.

Which of the following characteristics would not define an organization as governmental? A)The power to enact and enforce a tax levy. B)The potential for unilateral dissolution by a government with the net assets reverting to a government. C)The receipt of grant money from a state or local government for the purpose of providing services to the public. D)All of the above.

C)The receipt of grant money from a state or local government for the purpose of providing services to the public.

Which of the following would be accounted for as a permanent fund?

C. A gift of $500,000 to a city, to be invested permanently, with the proceeds to be used to maintain the city cemetery.

Which of the following bond types has a payment schedule in which the amount of annual principal repayment is scheduled to increase each year by approximately the same amount that interest payments decrease?

C. Annuity

Which of the following best describes how many funds a government should use? A. One for each revenue source B. Two at a minimum; the General Fund and one enterprise fund C. As many as necessary to fulfill legal requirements and sound financial administration D. None of the above

C. As many as necessary to fulfill legal requirements and sound financial administration

A special assessment tax is:

C. Assessed only against certain property owners who are deemed to benefit from the service or project being paid for by the proceeds of the special assessment levy

In order for a fund to exist, there must be: A. A fiscal entity; assets set aside for a specific purpose B. A double-entry accounting entity C. Both A & B are required D. A & B are optional and not required

C. Both A & B are required

Which of the following is one of the criteria to determine if a governmental fund is considered to be a major fund? A. Total assets, liabilities, revenues or expenditures of the individual governmental fund constitutes 10 percent of the governmental funds category. B. Total assets, liabilities, revenues or expenditures/expenses are 5 percent of the total of the governmental and enterprise category. Other funds may be designated major funds at the discretion of management. C. Both A and B are required D. Either A or B will qualify a fund to be a major fund

C. Both A and B are required

With respect to the preparation of fund basis financial statements, governmental funds other than the General Fund are considered to be major when which of the following conditions exist? A. With respect to fund basis financial statements, governmental funds are considered to be a major fund when total assets, liabilities, revenues, or expenditures of that individual governmental fund constitutes 10% of the total for the governmental fund category. B. With respect to fund basis financial statements. governmental funds are considered to be a major fund when total assets, liabilities, revenues, or expenditures of that individual governmental or enterprise fund are 5% of the total of the governmental and enterprise categories, combined. C. Both A and B are required for a governmental fund to be a major fund. D. Either A or B would fulfill the requirements.

C. Both A and B are required for a governmental fund to be a major fund.

Encumbrance accounting is only typically used for:

C. Capital Project Funds

A construction contract was issued for n addition to the county jail in the amount of $2,100,000. When the project was approximately half finished, the contractor submitted an invoice for payment of $1,050,000. Which partial entry below is not required to record this transaction?

C. Credit Encumbrances Control $2,100,000

The journal entry for the Debt Service fund upon the receipt of money from the General fund would be:

C. Debit to Cash and credit to Transfer In.

Governmental-type funds label the excess of assets over liabilities on the balance sheet as: A. Net Assets B. Net Fund Balance and Liabilities C. Fund Balance D. Assets Net of Liabilities

C. Fund Balance

If taxes are levied specifically for payment of interest and principal of long term debt, those taxes are:

C. Included as revenues of the debt service fund

If a trust is to be used for the benefit individuals, private organizations or other governments, the following fund (s) should be used:

C. Private purpose fund

Which of the following is true with respect to long-term debt?

C. Resources transferred to the debt service fund from General Fund would typically be classified as Other Financing by the debt service fund.

Which of the following is a governmental fund? A. Agency B. Enterprise C. Special revenue D. Internal service

C. Special revenue

The GASB has the authority to establish accounting and financial reporting standards for: A. Federal government B. Private not-for-profits C. State governments D. None of the above

C. State governments

Which of the following statements are TRUE?

C. The principal of permanent funds are classified as Non spendable Fund Balance.

Which of the following statements regarding serial bonds is FALSE?

C. The principal repayment on an annuity serial bonds decreases each year as the outstanding balance decreases.

If the government is primarily or secondarily liable for the payment of debt principal and interest for a special assessment project:

C. The project is accounted for within governmental type funds as if it were a governmental project

Which of the following statements is false? A. With respect to fund basis financial statements, a government may designate any fund to be a major fund if reporting that fund separately would be useful. B. When preparing fund basis financial statements, any funds not reported separately are aggregated and reported in a single column under the label non-major funds. C. When preparing fund basis financial statements, any funds not reported separately are reported by function. D. In addition to the government-wide statements, governments are required to prepare fund financial statements for governmental, proprietary and fiduciary funds.

C. When preparing fund basis financial statements, any funds not reported separately are reported by function.

Which of the following is a major source of funding for capital project funds:

D. All of the above. Gifts from individuals and corporations Proceeds from issuance of Long Term Debt Grants

GASB requires which of the following to be reported separately after other financing sources and uses in the Statement of Revenues, Expenditures, and Changes in Fund Balance? A) Special items. B) Extraordinary items. C) Interfund transfers. D) A and B

D) A and B

139. What would be the amount of expenditures recorded by the debt service fund for the fiscal year ended December 31, 2015? A) $320,000. B) $120,000. C) $ 90,000. D) $ 60,000.

D) $ 60,000.

The Township of Thomasville's General Fund has the following net resources at year end: • $77,000 of prepaid insurance • $375,000 rainy day fund approved by the township governing board with specific conditions for its use • $2,500 of supplies inventory • $61,000 state grant for snow removal • $150,000 contractual obligations for the purchase of equipment • $200,000 to be used to fund government operations in the future • Outstanding encumbrance of $80,000 for the purchase of furniture & fixtures What would be the total Assigned fund balance? A) $105,000 B) $455,000 C) $ 67,500 D) $ 80,000

D) $ 80,000

The tax agency fund of Eden County collected $2,000,000 for the Eden School District, $1,000,000 for the Village of Edenton, $1,200,000 for the Eden Park District, and $700,000 for Eden County. County General Fund employees handle the collections, and a 3 percent collection fee is charged all units except the county. The amount to be remitted to the county General Fund would be: A) $126,000. B) $147,000. C) $747,000. D) $826,000.

D) $826,000.

A budgetary comparison schedule is required for? A) The General Fund. B) Each special revenue fund that has a legally adopted annual budget. C) Enterprise funds. D) (A) and (B) only.

D) (A) and (B) only.

Which of the following would be found in an Enterprise Fund Financial Statement? A) Expenditure - Supplies. B) Reserve for Encumbrance. C) Appropriations. D) Depreciation Expense.

D) Depreciation Expense.

The journal entry to record an encumbrance would include: A)A debit to Encumbrance Control B)A debit to Budgetary Fund Balance - Reserve for Encumbrances C)A credit to Budgetary Fund Balance - Reserve for Encumbrances D)A and C would both be included in the journal entry

D) A and C would both be included in the journal entry

The general ledger journal entry in the General Fund to record encumbrances for the issuance of purchase orders would include: A) A credit to Encumbrance Control B) A debit to Expenditures Control C) A credit to Accounts Payable D) A credit to Budgetary Fund Balance-Reserve for Encumbrances

D) A credit to Budgetary Fund Balance-Reserve for Encumbrances

$60,000 of property tax owed to the city of Akron will not be collected within 60 days after year end. The year-end journal entry to record this information would include a: A) A credit to Revenues Control. B) A debit to Deferred Inflows - Property Taxes. C) A credit to Income Taxes Receivable. D) A debit to Revenues Control.

D) A debit to Revenues Control.

Purchase orders for items ordered by the General Fund totaled $ 205,000. Upon receipt, invoices for these items totaled $200,000. Which of the following will take place on the date the goods are received. A) An encumbrance is debited for $205,000. B) Budgetary Fund Balance -- Reserve for Encumbrances is credited for $200,000. C) An expenditure is credited for $205,000. D) A liability is credited for $200,000.

D) A liability is credited for $200,000.

When accounting for the General Fund, the Encumbrances Control account is credited when: A) A purchase order is approved. B) The budget is approved. C) An invoice is paid. D) A purchase order is filled or canceled.

D) A purchase order is filled or canceled.

Budgetary Fund Balance -- Reserve for Encumbrances (current year) in excess of a balance of Encumbrances Control indicates: A) An excess of Vouchers Payable over Encumbrances Control. B) An excess of purchase orders released over invoices received. C) An excess of invoices received over purchase orders released. D) A recording error.

D) A recording error.

A permanent fund classified under governmental funds... A) Accounts for most of the basic services provided by the governmental unit. B) Accounts for financial resources intended for major capital projects C) Accounts for services provided by one department of a government to another D) Accounts for resources that are legally restricted so only earnings, not principal, may be expended, and for purposes to benefit the government and its citizenry.

D) Accounts for resources that are legally restricted so only earnings, not principal, may be expended, and for purposes to benefit the government and its citizenry.

What account should be credited in a pension trust fund to record employee contributions? A) Other Financing Sources B) Net Assets C) Pension Benefit Obligation D) Additions

D) Additions

The ______ Funds are used to account for situations in which the government is acting as a collecting/disbursing agent. A) Special revenue B) General C) Enterprise D) Agency

D) Agency

Which of the Fiduciary Funds listed below is used to account for the assets held by a government acting as an agent for one or more other governmental units? A) Private-purpose Trust Fund. B) Permanent Fund. C) Pension Fund. D) Agency Fund.

D) Agency Fund.

According to GASB Statement 34, restricted net assets are those that are the result of constraints from: A) Imposed by law though constitutional provisions B) Imposed by enabling legislation C) Imposed by creditors or grantors D) All of the above

D) All of the above

According to GASB Statement No. 54 which of the following is an example of a nonspendable fund balance? A) Resources (such as supplies) that are used in operations rather than converted to cash B) The corpus of a permanent fund that may not be spent C) Prepaid insurance D) All of the above

D) All of the above

Which of the following expenditure classification schemes may be used by governmental funds A) Activity B) Character C) Function D) All of the above

D) All of the above

When governments operate landfills as enterprise funds, which of the following is correct? A) GASB requires that certain postclosure costs be estimated and accrued during the period the landfills receive solid waste. B) GASB requires that a portion of future estimated costs be charged as an expense and liability of the landfill using units-of-production method as waste is accepted. C) The purpose of recording postclosure expenses and liabilities is to match the estimated costs with the revenues during the period of time the waste is accepted. D) All of the above are correct

D) All of the above are correct

Which of the following is correct with respect to accounting for supplies inventories in a governmental fund? A) Supplies inventories may be recorded using either the purchase method or the consumption method. B) An amount equal to the ending balance of supplies is reported as Nonspendable Fund Balance. C) The consumption method is preferable over the purchase method since it requires no adjustment to supplies expense when preparing the government-wide statements. D) All of the above are correct

D) All of the above are correct

The "Hierarchy of GAAP" is established by the: A) Financial Accounting Standards Board. B) Federal Accounting Standards Advisory Board C) Governmental Accounting Standards Board D) All of the above have established a hierarchy

D) All of the above have established a hierarchy

GASB requires which of the following (if applicable) to be included in the Notes to Financial Statements? A) Outstanding encumbrances B) The definition of cash and cash equivalents used in the statement of cash flows for proprietary funds. C) Interfund receivables and payables. D) All of the above.

D) All of the above.

According to GASB 34, Enterprise funds must be used when: A) Debt is backed solely by fees and charges. B) A legal requirement exists that the cost of providing services for an activity, including capital costs, be recovered through fees or charges. C) A government has a policy to establish fees and charges to cover the cost of providing services for an activity. D) Any of the above apply.

D) Any of the above apply.

Revenues are raised to finance governmental activities, but revenues may be expended only for purposes and in amounts approved by the legislative branch in compliance with laws of competent jurisdictions. This process is known as the A) Budgetary Fund B) Encumbered Process C) Expended Process D) Appropriations Process

D) Appropriations Process

If Budgetary Fund Balance was debited in the process of recording a budget for the General Fund of a governmental unit, it can be assumed that: A) Estimated Revenues exceed Appropriations. B) Estimated Expenses exceed Actual Revenues. C) Actual Expenses exceed Estimated Expenses. D) Appropriations exceed Estimated Revenues.

D) Appropriations exceed Estimated Revenues.

Restricted Net Assets for a proprietary fund: A) Are net resources whose use is restricted by law B) Are reported net of depreciation C) Are reported net of accumulated depreciation and any outstanding debt used to acquire the asset D) Are net resources whose use is restricted by creditors, grantors or other governments.

D) Are net resources whose use is restricted by creditors, grantors or other governments.

Use the following to answer the next six questions: During the fiscal year ended December 31, 2015, the City of Johnstown issued 6% general obligation serial bonds in the amount of $2,000,000 at 102 ($2,040,000) and used $1,980,000 of the proceeds to construct a fire station. The $40,000 premium was transferred to a debt service fund. The $20,000 left in the capital projects fund at the end of the project was later transferred to the debt service fund. The bonds were dated April 1, 2015 and paid interest on October 1 and April 1. The first of 10 equal annual principal payments was due on April 1, 2016. 135. How would the bond sale be recorded? A) As a liability in the debt service fund. B) As a liability in the capital projects fund. C) As an other financing source in the debt service fund. D) As an other financing use in the capital projects fund.

D) As an other financing use in the capital projects fund.

The journal entry to record the prior year's deferred Inflows: property taxes (those expected to be collected more than 60 days beyond year-end) as revenue in the current year would include: A) A debit to Revenues Control B) A debit to Deferred Inflows: Revenues C) A credit to Revenues Control D) B and C would both be included in the journal entry

D) B and C would both be included in the journal entry

Which of the following is true of the Statement of Net Assets for proprietary funds? A) a classified format is used in which current assets, noncurrent assets, current liabilities, and noncurrent liabilities are presented separately. B) separate disclosure is required for major enterprise funds and a total for all non-major enterprise funds C) Neither A nor B are correct. D) Both A and B are correct.

D) Both A and B are correct.

Which of the following is most correct with regard to Management's Discussion and Analysis (MD&A)? A) State and local governments are required to provide an MD&A. B) Federal agency financial reports are required to provide an MD&A. C) Both state and local governments and federal agencies are encouraged, but not required to provide an MD&A. D) Both state, local governments, and federal agencies are required to provide an MD&A.

D) Both state, local governments, and federal agencies are required to provide an MD&A.

Indicate which of the following would not be an example of an internal service fund: A) City Self-Insurance Fund B) City Print Shop. C) City Data Processing Service Center. D) City Airport.

D) City Airport.

The estimated costs of closure of solid waste landfills are measured using: A) Estimated future costs. B) Actual closure costs. C) Historical costs. D) Current costs.

D) Current costs.

A pension plan that is required to pay out only the amount that has been accumulated for each employee is a ________? A) Defined Benefit Plan. B) Private-Purpose Plan. C) Defined Withdrawal Plan. D) Defined Contribution Plan.

D) Defined Contribution Plan.

If a government decides to account for its risk management activities in a single fund, it must use: A) An Internal Service Fund. B) A Fiduciary Fund C) A Special Revenue Fund. D) Either the General Fund or an Internal Service fund.

D) Either the General Fund or an Internal Service fund

How would purchase orders and contracts be reported under modified accrual accounting and accrual accounting? Modified Accrual Accrual A) Expenditure Liability B) Expenditure No entry C) Encumbrance Liability D) Encumbrance No entry

D) Encumbrance No entry

What type of fund is used when resources are provided mainly through business-like transactions to parties external to the government? A) Internal Service Funds B) Capital Projects Funds C) Agency Funds D) Enterprise Funds

D) Enterprise Funds

The Expenditures control account in the General Fund is debited when: A)Supplies are ordered. B)The budget is recorded. C)The books are closed at the end of the year. D)Equipment previously ordered is received.

D) Equipment previously ordered is received.

When liabilities authorized by an appropriation have been incurred, the appropriation is said to be... A) Expensed B) Encumbered C) Expired D) Expended

D) Expended

Liabilities incurred, such as for accrued payroll, but still unpaid at year-end, should be recorded in the General Fund by a debit to which one of the following accounts? A) Current Expense. B) Appropriations. C) Encumbrances. D) Expenditures.

D) Expenditures.

Wages that have been earned by the employees of a governmental unit, but not paid at year-end, should be recorded in the General Fund by a debit to which of the following accounts? A) Appropriations. B) Encumbrances. C) Expenses. D) Expenditures.

D) Expenditures.

At year-end, the balance of Expenditures, Revenues and Other Financing Sources and Uses accounts are closed to: A) Budgetary Fund Balance B) Expenses C) Expenditure Control D) Fund Balance

D) Fund Balance

When closing out the General Fund and Special Revenue Funds of a state or local governmental unit, the balance of the operating statement are closed to: A) Budgetary Fund Balance - Reserved for Encumbrances. B) Appropriations Control. C) Other Financing Sources - Transfer Out. D) Fund Balance.

D) Fund Balance.

Which of the following funds requires a Budgetary Comparison Schedule? A) Enterprise Funds B) Internal Service Funds C) Investment Trust Funds D) General Fund

D) General Fund

A city government sells police cars no longer in use. No restrictions have been placed on the proceeds. Which fund should account for the receipt? A) Debt service fund B) Capital projects fund C) Enterprise fund D) General fund

D) General fund

Capital assets of an enterprise fund should be reported in the: A) Government-Wide Statement of Net Assets only. B) Proprietary Fund Statement of Net Assets only. C) General Fixed Asset Account Group only. D) Government-Wide Statement of Net Assets and Proprietary Funds Statement of Net Assets.

D) Government-Wide Statement of Net Assets and Proprietary Funds Statement of Net Assets.

Which of the following statements regarding employer reporting of pension or other employee benefit trust funds is not correct? A) Contributions by the governmental funds are recorded as expenditures. B) Unfunded amounts are reported in the government-wide statements. C) Proprietary funds record an expense equivalent to the required contribution. D) Governmental funds report unfunded amounts as a fund liability.

D) Governmental funds report unfunded amounts as a fund liability.

The Revenues Control account of the General Fund is debited when: A) The budget is recorded at the beginning of the year. B) Uncollectible taxes receivable accounts are written off. C) Property taxes are collected. D) None of the above.

D) None of the above.

Which of the following is false regarding proprietary fund accounting? A) The economic resources measurement focus and accrual accounting are used. B) Capital assets are recorded in the accounts and depreciated over their useful lives. C) The Statement of Net Assets must be in a classified format, with current assets separated from noncurrent assets and current liabilities separated from noncurrent liabilities. D) In the Statement of Net Assets, the net assets are reported as either reserved or unreserved.

D) In the Statement of Net Assets, the net assets are reported as either reserved or unreserved.

Nonreciprocal interfund activity A)Includes interfund services provided and used B)Includes exchange-like transactions C)Includes interfund loans D)Includes interfund transfers and reimbursements

D) Includes interfund transfers and reimbursements

Which of the following is a proprietary fund? A) Investment trust B) Permanent C) Special revenue D) Internal service

D) Internal service

Centralized purchasing, computer services, and janitorial services are examples of activities that are commonly reported in: A) Enterprise funds. B) Capital project funds. C) Debt service funds. D) Internal service funds.

D) Internal service funds.

GASB Statement 31, Accounting and Financial Reporting for Certain Investments and for External Investment Pools, does not apply to which of the following investment types? A) External investment pools. B) Open-end mutual funds. C) Debt securities. D) Investments of pension funds.

D) Investments of pension funds.

67. Which of the following is not one of the four classes into which nonexchange transactions apply in accordance with GASB Statement No. 33? A) Voluntary nonexchange transactions. B) Imposed nonexchange revenues. C) Derived Tax Revenues. D) Mandatory exchange transactions.

D) Mandatory exchange transactions.

How are Fiduciary Funds presented in the Government-wide Financial Statements? A) Combined with Internal Service Funds. B) Combined with Business activities. C) Combined with governmental activities. D) None of the Above.

D) None of the Above.

Which of the following funds is used to account for the payment of principal and interest of general long term debt of a government? A) Capital Projects Fund B) Internal Service Fund C) Agency Fund D) None of the above

D) None of the above

The Budgetary Fund Balance -- Reserve for Encumbrances account is properly considered to be a(an): A) current liability. B) expenditure. C) long-term liability. D) None of the above.

D) None of the above.

The Water Utility (an enterprise fund) has just sent out a purchase order to a vendor for some new equipment. Which of the following would be included in the journal entry to record this event? A) A credit to Accounts Payable. B) A debit to Encumbrances. C) A debit to Expenditures. D) None of the above.

D) None of the above.

Governmental Funds Fund Balance is displayed in which of the following categories? A) Nonspendable, Spendable, Assigned, Unassigned B) Nonspendable, Restricted, Unrestricted C) Restricted, Unrestricted, Assigned, Unassigned D) Nonspendable, Restricted, Committed, Assigned, Unassigned

D) Nonspendable, Restricted, Committed, Assigned, Unassigned

The Statement of Cash Flows for a proprietary fund would include which of the following? A) Operating, investing, financing B) Operating, capital and related financing, investing C) Operating, investing, fiduciary and capital related financing activities D) Operating, investing, capital and related financing and noncapital related financing

D) Operating, investing, capital and related financing and noncapital related financing

A fund that exists when a government is the trustee for a defined benefit pension plan, or a defined contribution pension plan is a(n): A) Agency fund B) Private-Purpose Trust Fund C) Investment Trust Fund D) Pension Trust Fund

D) Pension Trust Fund

Which of the following funds is considered a Fiduciary Fund? A) Debt Service Fund B) Enterprise Fund C) Internal Service Fund D) Pension Trust Fund

D) Pension Trust Fund

______ funds are created when individuals or organizations contribute resources with the agreement that the income will be used to the citizenry or the government's programs. A) Agency B) Investment trust C) Private-purpose trust D) Permanent

D) Permanent

Which of the following is not a fiduciary fund? A) Private-Purpose Trust Fund. B) Investment Trust Fund. C) Pension Trust Fund. D) Permanent Fund.

D) Permanent Fund.

Jasper City is trustee for the Henry J. Moneybags Endowment Fund, created to provide scholarships for students. This is an example of which type of fund? A) Permanent. B) Pension trust. C) Agency. D) Private-purpose trust.

D) Private-purpose trust.

An example of expenditure classification by object would be: A) Current. B) Public safety. C) Police department. D) Salaries.

D) Salaries.

Financial reporting requirements for defined benefit pension plans include all of the following statements and schedules except? A) Schedule of Funding Progress. B) Statement of Changes in Plan Net Assets. C) Statement of Plan Net Assets. D) Schedule of Employee Contributions.

D) Schedule of Employee Contributions.

Short-term loans which are backed by the taxing power of the governmental unit and used to meet working capital requirements are called: A) Appropriation loan. B) Inter-fund loans. C) Other financing sources. D) Tax anticipation notes.

D) Tax anticipation notes.

What is the correct journal entry for a Tax Agency Fund to record tax levies of other governments certified to it? A) Taxes Receivable - Current Revenues Control B) Taxes Receivable - Current Due from Other Governments C) Taxes Receivable Transfer Out D) Taxes Receivable for Other Governments Due to Other Governments

D) Taxes Receivable for Other Governments Due to Other Governments

Which of the following is not true regarding enterprise funds? A) Enterprise funds record long-term debt directly in the fund accounts. B) Enterprise funds record capital assets directly in the fund accounts. C) Enterprise funds report a Statement of Cash Flows. D) The difference between assets and liabilities of enterprise funds is termed "Fund Balance."

D) The difference between assets and liabilities of enterprise funds is termed "Fund Balance."

When preparing financial statements, the internal portion of investment pools would be reported under which fund? A) Investment trust fund. B) Pension fund. C) General Fund. D) The fund(s) providing the resources.

D) The fund(s) providing the resources.

The City of Springfield has three pension plans: a locally administered police plan for which it is trustee, a statewide cost sharing plan, and a statewide agency plan. The City would include in its CAFR pension trust fund financial statements for: A) All three plans. B) The locally administered plan plus the statewide agency plan. C) Both statewide plans. D) The locally administered plan only.

D) The locally administered plan only.

Which of the following is not true regarding proprietary funds? A) Enterprise funds are used by governments to account for services provided to the general public on a user-charge basis. B) General obligation bonds that will be paid from enterprise revenues must be reflected in the accounts of enterprise funds. C) Internal service self-insurance should set fees based on anticipated charges or a long-range plan to break even over time. D) The operation of internal service funds has no impact on other funds because it is run as a business and provides services that would have been purchased elsewhere by the other funds.

D) The operation of internal service funds has no impact on other funds because it is run as a business and provides services that would have been purchased elsewhere by the other funds.

According to GASB Statement No. 54 which of the following statements is not true regarding assigned funds? A) Assigned fund balances represent governmental resources that the government intends to use for a specific purpose such as a debt service fund. B) Constraints on assigned resources are more easily changed than committed funds. C) For governmental funds other than the General Fund, this is the category for all positive residual fund balances. D) This is the only fund balance that can be negative

D) This is the only fund balance that can be negative

What are Enterprise funds used for? A) To account for pension and employee benefit funds for which the governmental unit is the trustee. B) To report resources that are legally restricted so only earnings, not principal, may be expended, and for purposes to benefit the government and its citizenry. C) To account for most of the basic services provided by the governmental units. D) To account for resources provided primarily through the use of sales and service charges to parties external to the government.

D) To account for resources provided primarily through the use of sales and service charges to parties external to the government.

All of the following are commonly used major revenue source classes except for A) Taxes B) Intergovernmental revenues C) Fines and Forfeits D) Transfers from other funds

D) Transfers from other funds

Long-term liabilities of an enterprise fund should be reported in the: Proprietary Fund Statements Government-Wide Statements A) No No B) No Yes C) Yes No D) Yes Yes

D) Yes Yes

Which of the following is an interfund transaction? A)Interfund services provided and used B)Interfund transfers C)Interfund loans D)All of the above are interfund transactions

D) all of the above are interfund transactions

Solid waste landfills are required to estimate the cost of closure which include: A) Cost of equipment used. B) Cost of landfill cover. C) Cost of caring for the site for 30 years. D) all of the above.

D) all of the above.

The General Fund of the City of X passed a budget, providing for $2,000,000 in anticipated revenues and $1,990,000 in anticipated expenditures. The journal entry(s) to record the budget would result in a: A) credit to Appropriations Control in the amount of $1,990,000. B) debit to Estimated Revenues Control in the amount of $2,000,000. C) credit to Budgetary Fund Balance in the amount of $10,000. D) all of the above.

D) all of the above.

Funds other than the General Fund must be considered a major fund when A)total assets, liabilities, revenues, or expenditures/expenses of that fund constitute 10 percent of either the governmental or enterprise category. B)total assets, liabilities, revenues, or expenditures/expenses of that fund are 5 percent of the total of the governmental and enterprise category. C) conditions of either A or B exist. D) conditions of both A and B exist.

D) conditions of both A and B exist.

In a budgetary entry (combining entry), if Estimated Revenues Control exceeds Appropriations Control, the excess would be: A) credited to Fund Balance-Unreserved. B) debited to Fund Balance-Unreserved. C) debited to Budgetary Fund Balance. D) credited to Budgetary Fund Balance.

D) credited to Budgetary Fund Balance.

Which of the following is/are true regarding Capital Projects Funds?

D. All of the above are true A major source of funding for capital projects is the issuance of long term debt Capital project funds only exist for the duration of the project for which it is created Capital Project funds used the Modified Accrual Basis of Accounting

Governmental funds use the: A) economic resources measurement focus and accrual basis of accounting. B) current financial resources measurement focus and accrual basis of accounting. C) economic resources measurement focus and modified accrual basis of accounting. D) current financial resources measurement focus and modified accrual basis of accounting

D) current financial resources measurement focus and modified accrual basis of accounting

State and local governmental funds are organized into three categories including: A) governmental, proprietary, and restricted B) proprietary, fiduciary, and restricted C) governmental, fiduciary, and restricted D) governmental, proprietary, and fiduciary

D) governmental, proprietary, and fiduciary

Revenues of governmental fund types should be recognized when: A) collected in cash. B) authorized by the budget ordinance. C) taxes become delinquent. D) measurable and available.

D) measurable and available.

The term "fiduciary funds" applies to: A) enterprise and internal service funds. B) pension, investment trust and enterprise funds C) enterprise, internal service, and private-purpose trust funds. D) none of the above answers are correct.

D) none of the above answers are correct.

A Budgetary Comparison Schedule is: A) optional under GASB standards for all funds. B) required by GASB for internal management reports only; not permitted for external financial reporting. C) required for all governmental fund types. D) required for the General Fund, and for special revenue funds, for which an annual budget has been adopted.

D) required for the General Fund, and for special revenue funds, for which an annual budget has been adopted.

Governmental funds include: A)Special revenue funds B)Internal service funds C)Debt service funds D)A and C

D)A and C

Which of the following is considered Required Supplementary Information (RSI)? A)Management's Discussion and Analysis. B)Budgetary Comparison Schedule. C)Schedule of Risk Management Activities. D)All of the above are considered RSI.

D)All of the above are considered RSI.

Which of the following is true regarding GASB's definition of the financial reporting entity? A)The financial reporting entity might include a primary government, component units, joint ventures, or a jointly governed organization. B)Primary governments may be general-purpose governments (such as states, cities, and counties) or special-purpose governments that have separately elected governing bodies, are legally separate and are fiscally independent of other state or local governments. C)Blending is used to incorporate component units when those component units are, in substance, part of the primary government. D)All of the above are true.

D)All of the above are true.

Which of the following is true regarding the government-wide financial statements? A)The government-wide financial statements are not required to present prior-year data. B)The government-wide financial statements include a Statement of Net Position, Statement of Activities, but not Statement of Cash Flows. C)The government-wide financial statements include capital assets, including infrastructure, and reflect depreciation, except for infrastructure using the modified approach. D)All of the above are true.

D)All of the above are true.

Which of the following is true regarding the governmental fund financial statements? A)The governmental fund financial statements include the Balance Sheet, and the Statement of Revenues, Expenditures, and Changes in Fund Balances. B)The governmental fund financial statements are prepared on the current financial resources measurement focus and modified accrual basis of accounting. C)The governmental fund Balance Sheet reflects the difference of assets minus liabilities as Fund Balance. D)All of the above are true.

D)All of the above are true.

Which of the following is included in Required Supplementary Information (RSI)? A)Management's Discussion and Analysis (MD&A). B)Budgetary Comparison Schedule. C)The Schedule of Funding Progress and Schedule of Employer Contributions when the government reports a pension trust fund. D)All of the above.

D)All of the above.

Which of the following items would likely appear in the reconciliation between the governmental fund financial statements and the governmental activities column in the government-wide financial statements? A)The inclusion of long-term liabilities in the government-wide financial statements and not in the governmental fund financial statements. B)The inclusion of depreciation of capital assets in the governmental activities column of the government-wide financial statements and not in the governmental fund statements. C)The additional accrual of some revenues and expenses in the government-wide financial statements that were not made in the governmental fund financial statements. D)All of the above.

D)All of the above.

Which of the following is true regarding the Budgetary Comparison Schedule? A)The Budgetary Comparison Schedule compares the actual results to the original budget, but display of variances is optional. B)The Budgetary Comparison Schedule must be prepared for the General Fund and each major special revenue fund that has a legally adopted budget. C)The Budgetary Comparison Schedule is considered part of the basic financial statements. D)Both A and C are true.

D)Both A and C are true.

The Town of Little River expects to collect $90,000 in sales tax from the state government within 30 days of the end of fiscal year 2015 for retail sales taking place in fiscal year 2015. What entry, if any, would Little River make at the end of 2015? A)Taxes receivable - current 90,000 Deferred Inflows 90,000 B) Taxes receivable - deferred 90,000 Revenues control 90,000 C) Taxes receivable - current 90,000 Due from state government 90,000 D)Due from state government 90,000 Revenues control 90,000

D)Due from state government 90,000 Revenues control 90,000

Which of the following is true regarding fiduciary fund financial statements? A)Fiduciary fund financial statements include the Statement of Fiduciary Net Assets, the Statement of Changes in Fiduciary Net Assets, and the Statement of Cash Flows prepared on the direct method. B)Fiduciary fund financial statements are prepared on the current financial resources measurement focus and modified accrual accounting. C)Both of the above. D)Neither of the above.

D)Neither of the above.

Which of the following is true regarding the government-wide Statement of Activities? A)General revenues include items such as charges for services and program fees. B)Fiduciary activities are included. C)Both of the above. D)Neither of the above.

D)Neither of the above.

Which of the following is true regarding the governmental fund statements? A)The governmental fund statements include a Statement of Cash Flows prepared on the direct method. B)The governmental fund statements are prepared using the economic financial resources measurement focus and modified accrual accounting. C)Both of the above. D)Neither of the above.

D)Neither of the above.

Which of the following is false regarding major fund reporting for governmental funds? A)The General Fund is always a major fund. B)Each fund that is considered major must be reported in a separate column in the governmental funds financial statements. C)A government may designate any fund as major if it feels that reporting that fund in the basic financial statements would be useful. D)None of the above, all are true.

D)None of the above, all are true.

Which of the following accounts of a governmental unit's General Fund is credited to close it at the end of the fiscal year? A)Appropriations Control. B)Other Financing Sources-Transfers In. C)Budgetary Fund Balance -- Reserve for Encumbrances. D)None of the above.

D)None of the above.

The Governmental Accounting Standards Board has been given authority to establish accounting and financial reporting standards for: A)all governmental units and agencies. B)federal, state, and local governments and governmentally-owned utilities, authorities, hospitals, and colleges and universities.. C)all governmental units and all not-for-profit organizations. D)state and local governmental entities, and governmentally-owned utilities, authorities, hospitals, and colleges and universities.

D)state and local governmental entities, and governmentally-owned utilities, authorities, hospitals, and colleges and universities.

The total fund balances presented in Governmental Funds Statements are different from the total net assets on the government-wide Statement of Net assets. Which of the following is not a reason for this difference? A. The two statements have different bases of accounting B. Capital assets used in government operations are not financial resources and therefore are not reported in the funds C. Some liabilities are not due and payable in the current period and are not reported in fund liabilities D. Accrued Wages Payable at year end are reported on the statement of net assets but are not reported in the fund statements

D. Accrued Wages Payable at year end are reported on the statement of net assets but are not reported in the fund statements

In addition to the government-wide statements, governments are required to prepare fund financial statements for which of the following category of funds? A. Proprietary funds B. Governmental type funds C. Fiduciary funds D. All of the above

D. All of the above

Which of the following is a common source of funding for capital projects?

D. All of the above are sources of funding for capital projects Issuance of long term debt Proceeds of dedicated taxes Grants from other governments

The following are sources of funds for the debt service fund except:

D. All of the above are sources of funds for the debt service fund Tax levies specifically designed for debt service. General taxes levied by the General Fund and transferred to the debt service fund. Special assessments levied against property

A government entered into a capital lease agreement to acquire equipment for the general government on Jan 1 2014. Five payments of $8,000 each are to be made beginning on Dec. 31, 2014. Discounting is at 6% computed annually. The present value of the five payments is $33,699. Which of the following would be true as of Jan. 1, 2014?

D. An entry would be made debiting Expenditures and crediting Other Financing Sources - Capital Leases in a governmental fund, both in the amount of 33,699

If bonds issued to fund capital project are sold at a premium, the additional money received is:

D. Both A and B are correct A. Recorded as Other Financing Sources - Premium on Bonds in the capital projects fund. B. Transferred from the capital projects fund to the General fund.

At the inception of a capital lease agreement for a piece of equipment used in governmental operations:

D. Both B an C B. A liability is incurred and reported in the government-wide statements C. An expenditure is recorded in the governmental fund.

Governmental funds, other than the General Fund are considered major if:

D. Both I and II I - Total Assets, Liabilities, Revenues, or Expenditures of that individual governmental fund are at least 10% of the corresponding total ( assets, liabilities, and so forth) for all governmental funds. II - Total Assets, Liabilities, Revenues, or Expenditures of the individuals governmental fund are at least 5% of the corresponding total for all governmental and enterprise funds combined.

What is the correct debit or credit for the following scenario: A contract was issued for the major part of work to be done by a private contractor in the amount of $1,200,000 for a new County court house?

D. Debit: Encumbrances

A government signed a fist year capital lease on Jan 1. 2014 to obtain some equipment. The lease provided that the government would make a down payment of $10,000 and four $10,000 payment each year after that, beginning Jan. 1. 2014. The government has a fiscal year ending Dec. 31. Upon inception of the lease, the government in its governmental fund accounting records would:

D. Debt expenditures for the present value of the payments ( including $10,000), credit cash for $10,000, and credit other financing sources for the difference between he $10,000 and the present value of the future payments.

Which of the following serial bonds has the first scheduled installment delayed for a period of more than 1 year after the date of the issue?

D. Deferred Serial Bonds

With respect to Permanent Funds, which of the following is NOT true?

D. Earnings in excess of expenditures must be added to principals corpus

Which of the following fund(s) should be used if resources are provided by a donor with the stipulation that they used for the benefit of the citizenry?

D. Either A or B, depending upon whether the principal must be maintained. A. Special Revenue Fund B. Permanent Fund

With respect to Debt Service Funds, which of the following is NOT correct:

D. Encumbrances are recorded to reflect principal payments that are expected to come due within the current accounting period.

Which of the following funds typically record budgets?

D. General Funds and special revenue funds

Which of the following is true with respect to special assessment levies?

D. If the government is not liable for the special assessment debt directly or through guarantee, the special assessment is accounted for in an agency fund.

With respect to the preparation of fund basis financial statements, governmental funds other than the major fund are considered to be major when which of the following conditions exist? A. With respect to fund basis financial statements, governmental funds are considered to be a major fund when total assets, liabilities, revenues, or expenditures of the individual governmental fund under consideration constitutes 25% of the total for the governmental fund category. B. With respect to fund basis financial statements, governmental funds are considered to be a major fund when total assets, liabilities, revenues, or expenditures of individual governmental or enterprise fund under consideration are 15% of the total of the governmental and enterprise categories, combined. C. Both A and B are required for a governmental fund to be a major fund. D. Neither A nor B are correct.

D. Neither A nor B are correct.

General fixed assets that are acquired with governmental fund resources are reported as assets in the:

D. None of the above

Where should a government report special assessment debt that the government is NOT liable for in anyway.

D. None of the above.

Proceeds of tax supported bonds are recognized in a capital projects fund as a(an):

D. Other Financing source.

Which of the following statements is FALSE?

D. Principal of permanent funds is classified as Committed Funds Balance

Which of the following is NOT correct regarding Government Trust Accounting?

D. Trust funds accounted for in a permanent fund use accrual accounting

An encumbrance in a capital project fund is created:

D. When a contract is signed or issued.


Ensembles d'études connexes

Quiz 6: JavaScript 3 and the DOM

View Set

Human Growth and Development module 8

View Set

Marketing Midterm Practice Questions

View Set

Adjective suffixes - ive, -y, -ous, -ful,- less, -able, -ible, -al

View Set

Chapter 14 Risk of Infectious and Communicable Diseases

View Set